etmr1912

July 1, 2018 | Author: Yorlan Cervantes Perez | Category: Adrenocorticotropic Hormone, Infants, Medical Specialties, Clinical Medicine, Diseases And Disorders
Share Embed Donate


Short Description

Download etmr1912...

Description

PRUEBAS SELECTIVAS 2012/13 CUADERNO DE EXAMEN EXAMEN TIPO MIR 19/12

ADVERTENCIA IMPORTANTE ANTES DE COMENZAR SU EXAMEN, LEA ATENTAMENTE LAS SIGUIENTES

INSTRUCCIONES

1.

MUY IMPORTANTE: Compruebe IMPORTANTE: Compruebe que este Cuaderno de Examen lleva todas sus páginas y no tiene defectos de impresión. Si detecta alguna anomalía, pida otro Cuaderno de Examen a la Mesa. Realice esta operación al principio, principio, pues si tiene que cambiar el cuaderno de examen posteriormente, se le facilitará una versión “0”, que no coincide con coincide con su versión personal en la colocación de preguntas y no dispondrá de dispondrá de tiempo adicional.

2.

El cuestionario se compone de 225 preguntas más 10 de reserva. Tenga en cuenta que hay 30 preguntas que están ligadas a una imagen. Todas las imágenes están colocadas en la hoja nal de este cuestionario.

3.

Compruebe que el número de versión de versión de examen que gura en su “Hoja de Respuestas”, coincide con coincide  con el número de versión que gura en el cuestionario. Compruebe también el resto de sus datos identicativos.

4.

La “Hoja de Respuestas” está nominalizada. Se dispone de tres ejemplares en papel au au-tocopiativo que deben colocarse correctamente para permitir la impresión de las contestaciones en todos ellos. Recuerde que debe rmar esta hoja.

5.

Compruebe que la respuesta que va a señalar en la “Hoja de Respuestas” corresponde al número de pregunta del cuestionario. Sólo se valoran las valoran las respuestas marcadas en la “Hoja de Respuestas”, siempre que se tengan en cuenta las instrucciones contenidas en la misma.

6.

Si inutilizara su “Hoja de Respuestas”, pida un nuevo juego de repuesto a la Mesa de Exa Exa-men y no olvide consignar olvide consignar sus datos personales.

7.

Recuerde que el tiempo de realización de este ejercicio es de cinco horas improrrogables y que están prohibidos prohibidos   el uso de calculadoras calculadoras   (excepto en Radiofísicos) y la utilización de teléfonos móviles, móviles, o de cualquier otro dispositivo con capacidad de almacenamiento de información o posibilidad de comunicación mediante voz o datos.

8.

Podrá retirar su Cuaderno de Examen, una vez nalizado el ejercicio y hayan sido recogidas las “Hojas de Respuestas” por la Mesa.

EXAMEN ETMR 19/12

1.

1. Si se tratara de un macroadenoma hiposario pro ductor de ACTH, el tratamiento de elección sería

Pregunta vinculada a la imagen nº 1. En la imagen puede ver la TC torácica realizada a un paciente por sospecha clínica de cáncer de pulmón. Indique los hallazgos que mejor se ajustan:

2. 3.

1. Se observa una masa pulmonar derecha y adeno patías paratraqueales izquierdas. izquierdas. 2. Se observa una masa pulmonar derecha, una adenopatía hiliar derecha y una adenopatía subcarinal. 3. Se observa una masa pulmonar izquierda y múlti ples adenopatías mediastínicas. mediastínicas. 4. Se observa una masa pulmonar derecha derecha y múlti ples adenopatías paratraqueales derechas. 5. Se observa una masa pulmonar derecha, sin adeadenopatías mediastínicas.

4. 5.

5. 2.

3.

Pregunta vinculada a la imagen 3. Paciente de 25 años de edad que sufre accidente de motocicleta. Es trasladado a Urgencias, y en el estudio radiológico se objetiva una fractura con-

Pregunta vinculada a la imagen nº 1. Si en el paciente anterior la masa pulmonar mide 4 cm de diámetro, indique cuál sería la estadicación clínico-radiológica de este paciente:

minuta de la región metáso-episaria distal de la

1. 2. 3. 4. 5.

1. Solicitar estudio preoperatorio. 2. Reducción cerrada e inmovilización inmovilización con férula  posterior. 3. Realización de una TAC para planicación qui rúrgica. 4. Ingreso del paciente, y vigilancia para descartar desarrollo de un síndrome compartimental. 5. Todas las anteriores anteriores son correctas.

tibia y del peroné derecho. ¿Cuál de las siguientes actitudes le parece correcta?

T2N2. T2N1. T1N0. T1N2. T1N1

Pregunta vinculada a la imagen 2. Mujer de 42 años, que durante el estudio de colelitiasis se solicita TAC abdominal, encontrando el hallazgo observado en la imagen. La paciente presenta como antecedentes personales la existencia de obesidad, diabetes tipo 2, hipertensión arterial y en la exploración física destaca un predominio abdominal de la grasa, cara de luna llena, estrías

6.

Pregunta vinculada a la imagen 3. En el caso presentado, parece clara la necesidad de una cirugía para restaurar la anatomía y la función. Varias técnicas son posibles, de manera aislada o combinada. En muchos casos es de uti-

rojo-vinosas, plétora facial y extremidades nas.

lidad realizar una jación externa inicial (que se

En relación a la imagen, señale cuál de las siguientes causas es menos probable como origen del cuadro:

mantiene unas 2 semanas) para reducir la fractura y permitir la mejoría de las partes blandas, antes

1. Administración exógena de glucocorticoides. 2. Hiperplasia bilateral suprarrenal productora de cortisol.

1. Clavo intramedular. 2. Cerclaje con agujas y alambres.

de la jación denitiva con:

3. Placas y tornillos. 4. Tracción transesquelética. 5. Prótesis total de tobillo.

3. Microadenoma hiposario productor de ACTH. 4. Tumor ectópico produc productor tor de ACTH. 5. Macroadenoma hiposario productor de ACTH.

7. 4.

la cirugía transesfenoidal. La administración de ketoconazol ketoconazol previa a la cirugía podría ser útil en este caso. La suprarrenalectomía bilateral puede ser una opción terapéutica en caso de hiperplasia suprarrenal bilateral productora de cortisol. La suspensión de forma paulatina de los glucocorglucocorticoides exógenos conllevaría la normalización de la morfología suprarrenal. En la etiología etiología podría plantearse plantearse la posibilidad de la expresión de receptores aberrantes en la suprarrenal.

Pregunta vinculada a la imagen nº 4.

Pregunta vinculada a la imagen 2. Paciente de 44 años que acude por hemianopsia bitemporal, motivo por el cual se solicita resonancia magnética que muestra el siguiente resultado

En relación a la patología que presenta la paciente, señale la opción que le parezca INCORRECT INCORRECTA: A:

-2-

EXAMEN ETMR 19/12

1.

1. Si se tratara de un macroadenoma hiposario pro ductor de ACTH, el tratamiento de elección sería

Pregunta vinculada a la imagen nº 1. En la imagen puede ver la TC torácica realizada a un paciente por sospecha clínica de cáncer de pulmón. Indique los hallazgos que mejor se ajustan:

2. 3.

1. Se observa una masa pulmonar derecha y adeno patías paratraqueales izquierdas. izquierdas. 2. Se observa una masa pulmonar derecha, una adenopatía hiliar derecha y una adenopatía subcarinal. 3. Se observa una masa pulmonar izquierda y múlti ples adenopatías mediastínicas. mediastínicas. 4. Se observa una masa pulmonar derecha derecha y múlti ples adenopatías paratraqueales derechas. 5. Se observa una masa pulmonar derecha, sin adeadenopatías mediastínicas.

4. 5.

5. 2.

3.

Pregunta vinculada a la imagen 3. Paciente de 25 años de edad que sufre accidente de motocicleta. Es trasladado a Urgencias, y en el estudio radiológico se objetiva una fractura con-

Pregunta vinculada a la imagen nº 1. Si en el paciente anterior la masa pulmonar mide 4 cm de diámetro, indique cuál sería la estadicación clínico-radiológica de este paciente:

minuta de la región metáso-episaria distal de la

1. 2. 3. 4. 5.

1. Solicitar estudio preoperatorio. 2. Reducción cerrada e inmovilización inmovilización con férula  posterior. 3. Realización de una TAC para planicación qui rúrgica. 4. Ingreso del paciente, y vigilancia para descartar desarrollo de un síndrome compartimental. 5. Todas las anteriores anteriores son correctas.

tibia y del peroné derecho. ¿Cuál de las siguientes actitudes le parece correcta?

T2N2. T2N1. T1N0. T1N2. T1N1

Pregunta vinculada a la imagen 2. Mujer de 42 años, que durante el estudio de colelitiasis se solicita TAC abdominal, encontrando el hallazgo observado en la imagen. La paciente presenta como antecedentes personales la existencia de obesidad, diabetes tipo 2, hipertensión arterial y en la exploración física destaca un predominio abdominal de la grasa, cara de luna llena, estrías

6.

Pregunta vinculada a la imagen 3. En el caso presentado, parece clara la necesidad de una cirugía para restaurar la anatomía y la función. Varias técnicas son posibles, de manera aislada o combinada. En muchos casos es de uti-

rojo-vinosas, plétora facial y extremidades nas.

lidad realizar una jación externa inicial (que se

En relación a la imagen, señale cuál de las siguientes causas es menos probable como origen del cuadro:

mantiene unas 2 semanas) para reducir la fractura y permitir la mejoría de las partes blandas, antes

1. Administración exógena de glucocorticoides. 2. Hiperplasia bilateral suprarrenal productora de cortisol.

1. Clavo intramedular. 2. Cerclaje con agujas y alambres.

de la jación denitiva con:

3. Placas y tornillos. 4. Tracción transesquelética. 5. Prótesis total de tobillo.

3. Microadenoma hiposario productor de ACTH. 4. Tumor ectópico produc productor tor de ACTH. 5. Macroadenoma hiposario productor de ACTH.

7. 4.

la cirugía transesfenoidal. La administración de ketoconazol ketoconazol previa a la cirugía podría ser útil en este caso. La suprarrenalectomía bilateral puede ser una opción terapéutica en caso de hiperplasia suprarrenal bilateral productora de cortisol. La suspensión de forma paulatina de los glucocorglucocorticoides exógenos conllevaría la normalización de la morfología suprarrenal. En la etiología etiología podría plantearse plantearse la posibilidad de la expresión de receptores aberrantes en la suprarrenal.

Pregunta vinculada a la imagen nº 4.

Pregunta vinculada a la imagen 2. Paciente de 44 años que acude por hemianopsia bitemporal, motivo por el cual se solicita resonancia magnética que muestra el siguiente resultado

En relación a la patología que presenta la paciente, señale la opción que le parezca INCORRECT INCORRECTA: A:

-2-

EXAMEN ETMR 19/12

Varón de 55 años de edad, con vida sedentaria, que hace el Camino de Santiago. Una semana an-

(ver imagen). ¿Cuál de las siguientes armaciones

le parece INCORRECTA en relación al manejo inicial del paciente?

tes de nalizar su peregrinaje, presenta un dolor

1. Determinación de prolactina, ya que podría orientarnos a un tratamiento médico en vez de quirúrgico. 2. Solicitaría un perl básico de hormonas hipo-

intenso en el pie izquierdo, sin traumatismo que él recuerde. El pie aparece hinchado pero es capaz de concluir su marcha con analgésicos, vendaje, y frío local. Acude a la consulta, 4 semanas después del inicio del dolor por no n o mejorar. mejorar. Se explora y se

sarias para descartar un posible décit antes del

realiza una radiografía que nos conrma nuestras

tratamiento quirúrgico.

sospechas diagnósticas de:

3. El tratamiento será quirúrgico en cualquier caso.

4. Solicitaría una campimetría reglada para facilitar la evaluación en el seguimiento. 5. El tratamiento se basará en la cirugía y los análo gos de somatostatina en cualquier caso.

8.

1. 2. 3. 4. 5.

Pregunta vinculada a la imagen nº 4. Durante la valoración inicial, el paciente comienza con cefalea brusca de gran intensidad, int ensidad, nauseas, vómitos, disminución del nivel de conciencia, da-

11.

Pregunta vinculada a la imagen 6. El tratamiento a instaurar en este paciente, en el momento de la consulta consistirá en:

tos de irritación meníngea sin ebre, oftalmoplejia y alteraciones pupilares. ¿Cuál considera la ar-

mación CORRECTA entre entre las siguientes?

1. Disminución de actividad según dolor y esperar esperar a la resolución del cuadro.

1. Sospecharía un ACVA isquémico asociado a los décits de hormonas hiposarias

2. Cirugía para para reducción reducción y osteosíntesis.

3. Plantillas a medida para descarga descarga selectiva del  primer dedo. 4. Descarga de la extremidad y rehabilitación intensiva. 5. Los resultados de una resonancia resonancia magnética nos  permitirán establecer una estrategia de tratamien to.

2. Sospecharía una cefalea tensional asociado al estrés del diagnóstico de macroadenoma hiposario

3. Sospecharía un Síndrome de Sheehan por infarto isquémico de la hipósis. 4. Sospecharía una hipositis linfocitaria asociada a macroadenoma hiposario 5. Sospecharía un infarto hemorrágico en el seno del macroadenoma hiposario.

12. 9.

Esguince de Lisfranc. Fractura-luxación de Lisfranc. Fractura de la falange proximal del 2º dedo. Fractura de estrés del del 2º metatarsiano. Necesitamos una resonancia resonancia magnética para el diagnóstico.

Pregunta vinculada a la imagen 5.

Pregunta vinculada a la imagen 7. Mujer de 66 años que acude a consultas de endocrinología remitida por su médico de familia, al presentar alteraciones en la función tiroidea en la analítica rutinaria. En la exploración destaca la existencia de un bocio de consistencia irregular,

Paciente de 64 años, con antecedentes de hipertensión arterial en tratamiento, acude a su consulta reriendo sangrado vaginal de varios días

de evolución que actualmente es escaso. Usted decide practicarle una histeroscopia diagnóstica en la que observa los hallazgos que se muestran en la imagen. ¿Cuál sería su primera sospecha diagnóstica?

presentando en la analítica TSH: 0. 001 (N: 0.55 mcU/mL), T4 libre: 2.5 ng/dL (N: 0.7- 1.48), T3 libre: 6.70 pg/mL (N: 1.71-4.53). La paciente no

presenta sintomatología compresiva. Se solicita gammagrafía tiroidea, donde se observa la imagen previa. Señale el diagnóstico más probable. p robable.

1. Pólipo endometrial. 1. 2. 3. 4. 5.

2. Cáncer endometrio.

3. Mioma submucoso. 4. Endometrio atróco normal.

5. Útero septo.

10.

Enfermedad de Graves-Basedow Graves-Basedow.. Adenoma folicular hiperfuncionante. Tiroiditis silente. Tirotoxicosis facticia. Bocio multinodular hiperfuncionante.

Pregunta vinculada a la imagen 6. 13.

-3-

Pregunta vinculada a la imagen 7.

EXAMEN ETMR 19/12

o aire, pero si éstos no fueran efectivos sería necesaria la cirugía. 4. Realizar una manometría para apoyar la sospecha de Megacolon congénito. 5. Actitud expectante y revisión por su pediatra.

En relación a la patología que presenta la paciente, señale la opción CORRECT CORRECTA: A: 1. El tratamiento con antitiroideos no consigue la curación de la enfermedad. 2. Se trata de una enfermedad de naturaleza autoinmune. 3. El tratamiento habitual de elección es la tiroidectomía total.

16.

4. Es una patología rara en áreas áreas con décit de yodo. 5. La administración de radioyodo no está indicada,

Pregunta vinculada a la imagen nº 9.

ya que el hipertiroidismo se resuelve de forma es-

Un paciente de 23 años de edad asintomático acude a consulta por detectarle un ECG anómalo en

 pontánea.

un reconocimiento de empresa (ver imagen). El

paciente está asintomático y la exploración física es normal. Señale cuál de las siguientes opciones es verdadera: 14.

Pregunta vinculada a la imagen 8. 1. Se trata de un bloqueo completo de rama derecha. 2. Se trata de un bloqueo completo de rama izquierda.

Niño de 11 11 meses acude a urgencias por llanto intenso desde hace 2 horas. Los padres no ree ren ningún antecedente de interés salvo catarro de vías altas una semana antes sin ebre. Coincidiendo con los episodios de dolor retrae las piernas hacia el abdomen. A la exploración presenta regular estado general, con decaimiento importante y abdomen difícilmente depresible y doloroso, y durante la consulta emite una deposición en “jalea de grosellas”. Usted solicita una radiografía de abdomen que no muestra hallazgos relevantes por lo que se pide una ecografía abdominal. La imagen que aparece a continuación muestra lo que se observa en dicha ecografía. Respecto al cuadro clínico citado, ¿cuál de las siguientes aseveraciones es cierta?:

3. El ECG muestra muestra la presencia presencia de una vía accesoria. 4. El ECG muestra la presencia de una vía accesoria

oculta. 5. Exist Existee un RIV RIVA. A.

17.

Respecto al paciente anterior, señale la opción correcta en referencia a su tratamiento: 1. Dado que está asintomático se puede plantear se -

guimiento clínico sin empleo de ninguna medida terapéutica. 2. Es preferible el empleo de amiodarona por su

1. El cuadro es casi siempre secundario a una gastroenteritis infecciosa previa.

efecto modulador sobre el nodo AV.

3. En este paciente paciente es probable la presencia de IAM antiguo septal.

2. La prueba más sensible para el diagnóstico de

invaginación intestinal es la radiografía de abdomen. 3. La imagen muestra la característica imagen imagen del “donut”. 4. No es una situación de urgencia urgencia y se puede puede optar  por actitud expectante. 5. El primer diagnóstico de sospecha es el de vólvulo intestinal.

4. Los fármacos antiarrítmicos de de grupo Ic no son ecaces en el tratamiento.

5. Es necesaria una cardioversión cardioversión eléctrica urgente.

18. 15.

Pregunta vinculada a la imagen nº 9.

Pregunta vinculada a la imagen 10. Varón de 72 años que sufre accidente de tráfico con alcance posterior. Tras ser atendido en urgencias, los estudios de imagen - radiología simple, TAC, TAC, y resonancia magnética - determinan que sufre una fractura del cuerpo vertebral de C4, sin aparente inestabilidad ligamentosa. Adivine el tratamiento instaurado en este paciente:

Pregunta vinculada a la imagen 8. ¿Cuál sería el manejo más adecuado para la paciente de la pregunta anterior? 1. Lo probable es que se se trate de una estenosis estenosis hi pertróca de píloro por lo que debe realizarse una

 piloromiotomía de Ramsted. 1. Cementación vertebral y colocación de halo cra -

2. Se debe conrmar conrmar el diagnóstico con TAC TAC o RMN

abdominal. 3. El tratamiento inicial inicial en este caso es médico intentando la desinvaginación con enemas de suero

neal bloqueado. 2. Colocación de dispositivo tipo halo-chaqueta halo-chaqueta (o “halo-jacket”).

-4-

EXAMEN ETMR 19/12

4. La dermatitis de la interfase con daño de la mem brana basal orienta el diagnóstico histológico aun-

3. Reducción abierta y jación interna vertebral.

4. Manipulación vertebral holística y colocación de halo craneal.

que no es denitoria.

5. Las vesículas o ampollas son subepidérmicas.

5. Ortesis cráneo-facial-torácica.

22. 19.

Pregunta vinculada a la imagen 10.

Paciente de 62 años, DM tipo 2 de 12 años de evolución, en tratamiento con metformina 2000 mg/ día, sitagliptina 100 mg/día y gliclazida 60 mg/día. Como complicaciones vasculares conocidas presenta únicamente nefropatía diabética incipiente, con microalbuminuria positiva. En el último control presentaba una HbA1c de 7,8%. Acude hoy al Hospital de día de diabetes para revisión de posibles complicaciones vasculares. Dentro del estudio se obtiene la imagen 12 a nivel de la bifurcación carotídea. Señale la armación INCORRECTA:

En caso de no haber existido una lesión ósea, y haber sucedido una lesión ligamentosa con una clara inestabilidad segmentaria, el tratamiento hubiera consistido en: 1. Collarín cervical rígido.

2. 3. 4. 5.

20.

Pregunta asociada a la imagen 12.

Artrodesis vertebral. Prótesis de disco. Discectomía parcial y sutura ligamentosa. Plastia ligamentosa y rehabilitación intensiva.

1. El estudio de la íntima media carotídea debe hacerse a todos los pacientes diabéticos >50 años o 10 años de evolución. 2. La elección de la carótida se realiza por su accesi bilidad y para la detección de placas de ateroma, cuya presencia es más frecuente en las bifurcacio nes. 3. Se suele medir en la pared posterior. 4. Es una técnica no invasiva que permite estudiar la  progresión de la arterioesclerosis cuando todavía

Pregunta vinculada a la imagen nº 11. Una chica de 26 años consulta por la aparición de las lesiones observadas en la imagen, que se localizan en los codos, las rodillas y las nalgas. El cuadro tiene 6 meses de evolución y la paciente reere un

prurito intenso en las áreas afectadas y episodios diarreicos esporádicos. Responda la FALSA con respecto a esta patología:

no tiene signicación clínica.

1. Es habitual que se produzca afectación oral en forma de erosiones y, en raras ocasiones, ampollas tensas.

5. Se suelen realizar distintas mediciones a nivel de la carótida común, el bulbo carotídeo y carótida izquierda.

2. Característicamente, esta dermatosis no presenta

el signo de Nikolsky. 3. En una analítica sanguínea puede observarse un descenso de la ferritina y del calcio. 4. Es recomendable realizar un estudio endoscópico digestivo y la práctica de biopsias de mucosa in testinal. 5. Habitualmente los pacientes con esta dermatosis no tienen clínica digestiva.

21.

23.

Pregunta asociada a la imagen 12. En el paciente del caso anterior se completa el estudio realizándose las siguientes pruebas: índice tobillo-brazo mediante doppler, siendo el índice de 0,8. Con el monolamento de 10 gr. se calcula un índice de monolamentos de 6 puntos. Mediante el neuro-

tensiómetro se establece un umbral de percepción vibratoria de 20 mV. Desaparición de la sensibilidad vibratoria a nivel de 6 octanos con el diapasón. En la retinografía mediante retinógrafo digital se observa la presencia de microaneurismas y hemorragias en “llama”. Respecto a las conclusiones del estudio, señale la respuesta CORRECTA:

Pregunta vinculada a la imagen nº 11. En relación al aspecto histológico de las lesiones de la imagen, marque la que considere INCORRECTA: 1. El diagnóstico de sospecha se conrma mediante una biopsia y el estudio con inmunouorescencia

1. No se ha objetivado ningún dato de neuropatía.

2. El paciente presenta una retinopatía diabética proliferativa.

directa. 2. En el estudio histológico destaca la acumulación de polimorfonucleares en las papilas dérmicas. 3. Se produce una típica acumulación granular de IgA en las papilas de la dermis en la inmunouo rescencia directa.

3. Se ha objetivado enfermedad arterial severa. 4. El índice tobillo-brazo sugiere calcicaciones.

5. Es conveniente repetir la exploración oftalmológica cada tres años con el n de detectar una posi  ble progresión.

-5-

EXAMEN ETMR 19/12

24.

Pregunta vinculada a la imagen nº 13.

1. Mamograa bianual. 2. Mamograa anual. 3. Ecograa mamaria alternando con la mamograa

Varón de 52 años, que durante el estudio de colelitiasis se solicita TAC abdominal, encontrando el hallazgo observado en la imagen. El paciente presenta como antecedentes personales la existencia de obesidad, diabetes tipo 2, e hipertensión arterial en buen control, sin referir ninguna sintomatología de interés. ¿Cuál es el diagnóstico más probable que presenta el paciente?

 bianual. 4. Debido a los antecedentes familiares, deberia hacerse una RM. 5. Es indispensable la autoexploración mamaria periódica además de la mamograa para un buen

diagnóstico precoz.

1. Metástasis suprarrenal.

2. Feocromocitoma. 3. Adenoma suprarrenal no funcionante. 4. Tuberculosis.

28.

Pregunta vinculada a la imagen 15. Niño 6 meses con el antecedente de cuadro catarral

5. Carcinoma suprarrenal.

de 8 días de evolución, sin ebre, que su pediatra

del centro de salud ha tratado con mucolíticos y lavados nasales con suero siológico. En las últimas

25.

12 horas ha presentado varias crisis de llanto y un vómito, siendo las deposiciones normales, la última hace 24 horas. Durante su estancia en la sala de espera presenta una deposición con sangre fresca. A la exploración tiene todas las constantes normales para su edad, presenta un aceptable estado general, aunque usted la nota decaída, pálida y con tendencia al sueño. La auscultación cardíaca y pulmonar son normales. En la palpación abdominal no se evidencian masas ni megalias, el abdomen es blando y depresible, y no presenta signos de irritación peritoneal, aunque a usted le impresiona de dolor la palpación en hemiabdomen derecho. Tras la palpación abdominal la niña presenta una crisis de llanto agudo, con encogimiento de piernas de 2 minutos de duración, quedándose posteriormente tranquila y con tendencia al sueño. Decide pedir una ecografía urgente que puede observar en la imagen que acompaña a esta pregunta. Respecto al cuadro que sospecha, indique cuál de los siguientes enunciados le parece menos correcto:

Pregunta vinculada a la imagen nº 13. En relación a la patología que presenta el paciente, señale la opción CORRECTA: 1. Se encuentra indicada la determinación de catecolaminas, supresión con 1 mg de dexametasona y cociente ALD/ARP. 2. Se debe realizar suprarrenalectomía unilateral, valorando la administración de mitotane. 3. Dada la sospecha diagnóstica, se debe solicitar PAAF de la masa suprarrenal. 4. Se debe iniciar tratamiento sustitutivo con hidrocortisona, ya que probablemente desarrolle una insuciencia suprarrenal.

5. Se debe iniciar tratamiento inmediato con tuberculostáticos.

26.

Pregunta vinculada a la imagen 14. 1. Este cuadro clínico es la causa más frecuente de

Paciente de 54 años, con menopausia a los 49 años, fumadora de 15 cig/dia, y sin antecedentes médicos de interés. Antecedentes familiares: hermana diagnosticada de cáncer de mama a los 50 años. En la mamograa de cribado poblacional se ob serva la imagen que se muestra. Según el sistema BI-RADS esta mamografía se evaluará como: 1. 2. 3. 4. 5.

obstrucción intestinal entre los 3 meses y los 6 años de edad, con un pico de máxima incidencia

entre los 4 y los 12 meses, siendo raro en menores de un mes. 2. En la mayoría de los casos la causa es desconocida, aunque en mayores de 2 años es más probable

que exista una causa anatómica que desencadene el cuadro. 3. Algunas causas relativamente frecuentes en los casos secundarios son la púrpura de SchönleinHenoch, infecciones por adenovirus y el divertículo de Meckel. 4. La localización más frecuente es a nivel ileocóli co, por lo que en la exploración podemos encontrar una masa alargada y dolorosa en hipocondrio

BI-RADS 0. BI-RADS 1. BI-RADS 2. BI-RADS 3. BI-RADS 4.

derecho, con eje mayor cefalocaudal.

27.

Pregunta vinculada a la imagen 14.

5. En la ecografía abdominal, prueba diagnóstica de  primera elección, son típicos el signo de la rosquilla y el signo del muelle enrollado.

En la paciente del caso clínico anterior, ¿cuál sería el cribado recomendado?:

-6-

EXAMEN ETMR 19/12

29.

5. La mayoría de meningitis de causa otógena son causadas por  H. inuenzae.

Pregunta vinculada a la imagen 15. Al paciente de la pregunta anterior se le canaliza una vía intravenosa para analgesia y sueroterapia y el radiólogo realiza la reducción hidrostática de la invaginación con suero salino siológico controlada por ecografía. Respecto al tratamiento de este cuadro señale la correcta respecto a las indicaciones de tratamiento quirúrgico:

32.

1. Imposibilidad para la reducción hidrostática.

2. 3. 4. 5.

Signos de peritonitis. Signos de perforación intestinal. Neumatosis intestinal. Tiempo de evolución superior a 24 horas.

En la exploración de un paciente que acude a su consulta con desviación de la comisura labial a la derecha, lagoftalmos izquierdo y asimetría de arrugas frontales usted decide realizar una serie de pruebas diagnósticas entre las que encuentra un test de Schirmer alterado demostrando una hiposecreción lagrimal basal. Indique cuál de las siguientes patologías es más probable que haya desencadenado este cuadro clínico: 1. Cirugía de neurinoma del acústico.

30.

2. Otitis media crónica colesteatomatosa. 3. Otitis externa maligna. 4. Fractura transversal de peñasco.

Pregunta vinculada a la imagen 16. Paciente de 42 años, con menarquia a los 10 años, un embarazo y parto normal, con ciclos regulares y portadora de DIU de cobre como método de

5. Cirugía sobre la parótida.

planicación familiar. Antecedentes familiares:

madre intervenida de cáncer de mama bilateral, antes de los 50 años, que falleció a los 56 años sin realizarse estudio genético. Exploración mamaria dentro de la normalidad sin telorrea asociada. En una mamografía de seguimiento a los 6 meses por una mamografía anterior informada como BIRADS 3 se observa un aumento de las alteraciones observadas, siendo la mamografía actual la que se muestra en la imagen. ¿Cuál de las siguientes armaciones es correcta?:

33.

Para el cribado neonatal universal de la hipoacusia se emplean como primera opción en la mayoría de los centros hospitalarios: 1. Las otoemisiones acústicas. 2. La RMN funcional. 3. PEATC.

4. La electrococleografía. 5. La impedanciometría.

1. La mamografía se informará como BI-RADS 4.

2. Son hallazgos de características benignas. 3. Se recomiendan controles cada 6 meses durante 2 años. 4. Es necesaria una ecografía de mama complementaria para evaluar las lesiones observadas. 5. Deberemos realizar una galactografía para descartar un papiloma intraductal.

34.

Una paciente de 76 años diagnosticada recientemente de brilación auricular, presentó un infarto

cerebral hace 10 días. El examen neurológico de la paciente muestra una hemianopsia homónima derecha. La paciente no presenta alteración en la emisión o comprensión del lenguaje, pero no identica un reloj en la muñeca del examinador.

31.

Sin embargo, reconoce ese mismo objeto correctamente cuando se le presenta de forma aislada, sin necesidad de tocarlo. ¿Cómo denominaría el

Respecto a las complicaciones de las otitis medias, señale la respuesta correcta:

décit que presenta la paciente?

1. El síndrome de Gradenigo (otorrea + dolor retroo cular + diplopia) es la manifestación más caracte -

1. Ceguera cortical.

2. 3. 4. 5.

rística de la mastoiditis. 2. La petrositis es la complicación más frecuente.

3. La laberintitis purulenta produce lesiones irreversibles mientras que, en cambio, en la laberintitis serosa las lesiones son típicamente reversibles.

Metamorfopsia. Palinopsia. Prosopoagnosia. Simultagnosia.

4. Casi siempre se va a poder controlar el cuadro

35.

mediante ingreso hospitalario, tratamiento intravenoso y control clínico estricto, quedando relegada la cirugía para casos raros de alta gravedad.

-7-

Varón de 78 años de edad. Hipertenso en tratamiento con perindopril e indapami da desde hace 4 años, con buen control de cifras de tensión ar-

EXAMEN ETMR 19/12

2. El tratamiento de elección es la toxina botulínica. 3. No es una distonía ya que no se produce con otras actividades que involucran a la mano izquierda.

terial. Dislipemia en tratamiento con simvastatina 20 mg al día. No reere antecedentes de diabetes mellitus, no fumador. Bebedor ocasional. Intervenido de catarata en ojo derecho hace 9 meses, con buen postoperatorio, y recuperación de visión. Consulta por haber presentado en los tres últimos meses, cuadros bruscos de pérdida de visión, en ojo izquierdo, con desaparición completa de la misma y recuperación progresiva

Es un síntoma psico-somático.

4. Existe la posibilidad de que este paciente desarrolle otra distonía tarea especíca si cambia de

instrumento. 5. Probablemente esta alteración se produce por un cambio en la plasticidad cerebral en el área corti cal que controla la mano izquierda.

hasta la normalidad en unos 30 minutos. Reere

que esta situación le ha ocurrido en tres ocasiones. En una de ellas, se le acompañó de acorchamiento en mano derecha, que desapareció a la vez que la mejoría de la visión. Se ha realizado un TAC craneal que no ha demostrado lesiones en el momento actual. En relación a este paciente, ¿cuál de las siguientes armaciones conside ra correcta?

38.

1. Sin duda, la clínica que presenta este paciente se relaciona con patología de los medios ópticos, y

1. ACVA en territorio de la arteria cerebral posterior

izquierda.

requerirá intervención sobre su ojo izquierdo. 2. La normalidad en la prueba de imagen (TAC cra neal) nos permite excluir patología vascular cere -

2. ACVA en territorio de la arteria cerebral media

derecha.

 bral en este paciente. 3. El cuadro clínico corresponde a amaurosis fugax, y probablemente presente una estenosis signica tiva de carótida interna izquierda. 4. El cuadro clínico corresponde a amaurosis fugax, y probablemente presente una estenosis signica tiva de carótida interna derecha. 5. Puesto que ha presentado acorchamiento en mano derecha, estaría indicada una electromiografía de MSD para descartar neuropatía periférica.

36.

3. ACVA en territorio de la arteria cerebral media

izquierda. 4. ACVA en territorio de la arteria cerebral posterior

derecha. 5. Hemorragia subaracnoidea.

39.

¿Cuál es el tumor cerebeloso más frecuente en el adulto? 1. 2. 3. 4.

Acude a su consulta un paciente de 25 años, epiléptico desde la infancia, que se encuentra en tratamiento con carbamacepina. Desde hace unos 10 días se queja de diplopia e inestabilidad de la marcha. ¿Cuál de las siguientes armaciones con sidera falsa? 1. Probablemente se trate de síntomas producidos  por la carbamacepina. 2. El cuadro puede deberse a una interacción farmacocinética. 3. Hay que hacer una resonancia inmediatamente  para descartar un tumor. 4. Es preciso realizar unos niveles de carbamacepina. 5. Los niveles de carbamacepina probablemente es-

Hemangioblastoma. Astrocitoma pilocítico. Meduloblastoma. Neurinoma acústico.

5. Metástasis.

37.

Un paciente de 75 años es traído al servicio de Urgencias por presentar, de forma brusca, hemiparesia derecha de predominio braquial. A la exploración, muestra hemianopsia homónima derecha y disfasia uente con nula comprensión. El diagnóstico de sospecha es:

Paciente de 35 años, violinista profesional que nota que desde hace un par de meses mientras practica con el instrumento tiene una exión forzada del índice izquierdo que le impide tocar algunas notas. Esto no se produce cuando escribe o hace otra actividad. Respecto a este paciente señale lo falso:

tarán por encima del límite superior terapéutico.

40.

1. Probablemente es una distonía focal tarea especíca.

-8-

Un paciente de 29 años de edad, sin antecedentes personales de interés, ingresa en el servicio de Urgencias por un cuadro de cefalea, acompañado de náuseas y vómitos sin causa aparente, con adormecimiento progresivo del brazo derecho, hasta la paresia que el paciente presenta al llegar a Urgencias, no habiendo transcurrido en todo el proceso

EXAMEN ETMR 19/12

más de treinta minutos. Con el paso de los minutos el paciente está estuporoso. Al realizar un TAC craneal se aprecia la existencia de una hemorragia intraparenquimatosa. Pensando en una malformación vascular cerebral como primera causa del cuadro clínico, ¿cuál de las siguientes malformaciones vasculares cerebrales con producción de sintomatología es la más frecuente?

43.

Varón 60 años traído por su familia porque desde hace 4 meses se cae continuamente cuando va por la calle, caídas sin explicación alguna, ya que el paciente no se marea ni pierde fuerza. A la exploración llama la atención una facies inexpresiva con gran apertura de ojos, posición en hiperextensión del cuello, rigidez axial y ausencia de temblor de reposo. Cuál sería su idea diagnóstica inicial:

1. Angioma venoso. 2. Malformación arteriovenosa.

1. Parálisis Supranuclear Progresiva (PSP). 2. Enfermedad de Parkinson idiopática de predomi-

3. Angioma cavernoso (cavernoma).

nio rígido. 3. Encefalopatía espongiforme bovina.

4. Telangiectasias capilares. 5. Fístulas arteriovenosas durales.

4. Atroa Multisistémica.

5. Enfermedad de Huntington.

41.

Un varón de 30 años presenta un trastorno en la relajación de ambas manos, calvicie frontal y debilidad facial bilateral. El diagnóstico clínico es:

44.

1. Enfermedad de Becker. 2. Enfermedad de Steinert. 3. Distroa facio-escápulo-humeral. 4. Distroa muscular de las cinturas. 5. Distroa oculofaríngea.

Paciente de 20 años que presenta un cuadro progresivo de sordera bilateral e inestabilidad en la marcha. En el estudio de TC craneal muestra dos lesiones a nivel de ambos ángulos pontocerebelosos. ¿Cuál es el diagnóstico? 1. Neurobromatosis tipo I. 2. Neurobromatosis tipo II.

3. Esclerosis tuberosa. 4. Metástasis craneales.

5. Astrocitomas múltiples.

42.

Un paciente varón de 67 años de edad, hipertenso en tratamiento con enalapril 20 mg al día, y diabético tipo II en tratamiento con metformina 850 mg en desayuno, comida y cena, acude a la consulta contando un peculiar cuadro que consiste en para caminar, hormigueos en el brazo izquierdo y disartria, que característicamente se produce cuando ejercita su brazo izquierdo. Estos episodios le llevan ocurriendo desde hace unos tres meses. A la exploración física destaca una diferencia tensional de 40 mmHg a favor de la toma en el brazo derecho. En relación a la clínica de este paciente señale la respuesta correcta:

45.

Varón de 24 años de edad que acude a Urgencias por presentar cefalea intensa, discreta alteración de la conciencia y dicultad para hablar. En los antecedentes gura un accidente de tráco sufri -

do una hora antes con pérdida instantánea de la conciencia y recuperación total de la misma en uno o dos minutos, pudiendo atender a los demás heridos. Ha estado bien hasta hace media hora, en que comienza con cefalea intensa y dicultad

para hablar que va aumentando, así como una marcada tendencia al sueño, quedándose inconsciente en cuanto no recibe estímulos. Exploración física general sin hallazgos, aparte de un hematoma de cuero cabelludo en región fronto-temporal izquierda y fetor alcohólico. Exploración neurológica: hematoma fronto-temporal izquierdo, discreta obnubilación de la conciencia con tendencia al sueño (Glasgow 9), dudosa paresia facial derecha, anisocoria con pupila izquierda mayor que la derecha. Reejos fotomotores perezosos en am bos lados, discreta disminución de fuerza en MSD

1. Probablemente corresponde a disección de la arteria vertebral izquierda. 2. El paciente presenta claros síntomas de síndrome de túnel del carpo en su brazo izquierdo. 3. Probablemente la clínica corresponde a un síndrome de robo de la subclavia. 4. El estudio Doppler de arterias vertebrales, con  prueba de esgmomanómetro, no es de utilidad

 para el estudio de este paciente. 5. La diferencia en la toma de las tensiones se ex plica por una mala colocación del manguito del

(claudica antes en la maniobra de Mingazzini),

plantar derecho en extensión. Resto de la exploración sin hallazgos. ¿Cuál es la sospecha diagnóstica más probable?

esgmomanómetro.

-9-

EXAMEN ETMR 19/12

50.

1. Contusión cerebral. 2. Hemorragia subaracnoidea traumática.

3. Hematoma epidural agudo. 4. Hemorragia intraparenquimatosa. 5. Etilismo agudo.

La ausencia de coestimulación en el linfocito T induce en él: 1. Delección clonal. 2. Anergia clonal. 3. Citotoxicidad. 4. Producción de IFN-γ.

46.

5. Producción de IL-12.

Una vez identicados los problemas y necesidades de salud, el siguiente paso de la planicación de

salud es: 51. 1. La estimación de recursos. 2. La priorización de problemas. 3. La programación. 4. El análisis de los factores condicionantes. 5. La ejecución de actividades.

47.

¿Cuál de los siguientes isotipos de inmunoglobulinas está principalmente implicado en la defensa frente a bacterias encapsuladas?. 1. 2. 3. 4.

IgG4. IgG2. IgG3. IgG1.

5. Complemento.

¿En cuál de las siguientes fases de la planicación

sanitaria no se centra la contribución de la epidemiología? 52. 1. Identicación de problemas.

2. 3. 4. 5.

48.

Priorización de problemas. Determinación de actividades. Evaluación. Es útil en todas ellas.

pediatra pone de maniesto dicultad para ganar

peso. El diagnóstico más probable es: 1. 2. 3. 4.

En una población de 10.000 personas, de las cuales el 50% son mujeres y de éstas el 50% están en edad fértil, se realiza un estudio en el que la tasa de fecundidad fue de 32 por 1000 habitantes y la tasa de mortalidad de 25 por 1000 habitantes. ¿Cuál es la tasa de natalidad?

49.

Deciencia de IgA. Enfermedad de Bruton (agammaglobulinemia). Síndrome variable común de inmunodeciencia. Inmunodeciencia combinada y severa.

5. Síndrome hiper IgM.

53. 1. 2. 3. 4. 5.

Paciente de 18 meses que, desde hace un año, ha padecido múltiples episodios de otitis, sinusitis y tres episodios de neumonía. El seguimiento por el

40 por 1000 habitantes. 32 por 1000 habitantes. 8 por 1000 habitantes. 26 por 1000 habitantes. No se puede calcular con los datos suministrados.

¿Cuál es la principal clase de inmunoglobulina en la defensa de las mucosas? 1. 2. 3. 4. 5.

IgG. IgM. IgA. IgD. IgE.

¿Cuál de los siguientes no es un órgano linfoide secundario? 54. 1. Placas de Peyer. 2. Ganglios linfáticos.

3. Amígdalas palatinas. 4. Bazo. 5. Hígado.

-10-

Niño de 3 días que ha nacido de parto de nalgas y que presenta desde hace unas horas escasa reactividad y desconexión con el medio. En la exploración lo encuentra decaído, pálido, con tinte ictérico, taquicárdico y taquipneico. En el abdomen palpa una masa redondeada y lisa, en hipocondrio derecho, que parece depender del hígado. Señale la opción INCORRECTA acerca de la patología que sospecha:

EXAMEN ETMR 19/12

1. La prueba diagnóstica inicial debe ser un TC.

3. Labio superior no.

2. Si se rompe hacia la cavidad peritoneal puede producirse shock y la muerte del niño. 3. A veces el abdomen muestra un color azulado. 4. Los primeros datos suelen aparecer a partir del 3er-4º días de vida.

4. Retraso mental. 5. Hipoplasia de arco aórtico.

58.

5. Suele ser consecuencia de la presión que se ejerce

sobre el hígado durante la extracción de la cabeza en los partos de nalgas.

Señale lo que NO es correcto en cuanto al erisipeloide: 1. Es una enfermedad generalmente localizada que afecta a trabajadores que manejan pescado y/o

55.

carne. 2. No se han descrito casos de diseminación sistémica. 3. En la mayoría de los casos las lesiones aparecen

Neonato de 38 semanas de edad gestacional que en el paritorio presenta cianosis generalizada, tiraje intenso, aleteo nasal y rinorrea mucosa que se alivian parcialmente cuando el niño comienza a llorar. No se puede llevar a cabo la alimentación oral porque se vuelven a producir crisis de ciano-

en brazos y antebrazos y son más violáceas que

las de la celulitis y la erisipela. 4. A diferencia de la celulitis y la erisipela, generalmente carece de síntomas constitucionales y linfangitis. 5. El diagnóstico es fundamentalmente clínico y el tratamiento de elección es la penicilina.

sis y dicultad respiratoria intensas. ¿Cuál sería

su diagnóstico de sospecha? 1. Taquipnea transitoria del recién nacido. 2. Atresia tráqueo-esofágica.

3. Atresia de coanas. 4. Sílis congénita.

59.

5. Traumatismo nasal y hematoma del tabique tras el  parto.

56.

El nivel III de la clasicación de Clark en el mela-

noma maligno corresponde a una de las siguientes proposiciones. Indíquela:

En la enfermedad de membrana hialina o síndrome de distrés respiratorio neonatal existe un dé cit de surfactante que se ha relacionado con todas EXCEPTO una de las siguientes situaciones:

1. 2. 3. 4. 5.

Invasión de la epidermis. Invasión de la dermis papilar parcialmente. Invasión de la dermis papilar totalmente. Invasión de la dermis reticular parcialmente. Invasión de la hipodermis.

1. Prematuridad. 2. Hijo de madre diabética.

60.

3. Hipoxemia. 4. Trastorno metabólico de la síntesis de lecitina. 5. Hijos de madres adictas a heroína.

57.

Paciente mujer de 42 años de edad, con historia ginecológica de G3-P2-A1-HV2, con antecedente médico de alcoholismo crónico de 6-8 cervezas/ día, mantenido durante toda la gestación, da a luz a las 37 semanas de gestación a un recién nacido a término, de sexo femenino, talla de 48 cm, perímetro cefálico de 32 cm, perímetro torácico de 33 cm, con test de Apgar de 4 puntos al minuto y de 10 puntos a los 10 minutos, con maniobras de reanimación convencionales. En relación con el caso clínico. Señale la peculiaridad clínica que NO es característica del síndrome alcohólico fetal:

Un niño de dos años consulta por la aparición de una mácula marronácea, única, de bordes difusos. Al parecer, apareció cuando tenía pocos meses de vida. En la exploración física, la palpación revela que en realidad se trata de un nódulo, aunque la primera inspección revelaba una lesión maculosa. Usted se da cuenta de que, después del roce de la lesión al explorarla, se produce eritema, edema y prurito a nivel local, que vuelve a desaparecer a las pocas horas. El diagnóstico más probable sería: 1. Urticaria pigmentosa. 2. Enfermedad de Hand-Schüller-Christian. 3. Granuloma eosinólo.

4. Mastocitoma solitario. 5. Enfermedad de Letterer-Siwe.

61.

1. Bajo peso al nacimiento.

2. Microcefalia.

-11-

Varón de 52 años, asintomático hasta hace tres semanas, cuando comienza con deterioro rápida-

EXAMEN ETMR 19/12

4. PAR.

mente progresivo del estado general, molestias abdominales difusas, distensión abdominal progresi-

5. GPVI.

va y ebre ocasional. En la exploración física el

abdomen está distendido y difusamente doloroso. En TAC abdominal se aprecian grandes adenopatías mesentéricas y retroperitoneales, con engrosamiento mesentérico y moderada ascitis. Este patrón clínico es más característico del siguiente linfoma: 1. 2. 3. 4. 5.

62.

65.

Linfoma folicular. Linfoma no hodgkiniano difuso de célula grande. Linfoma de células del manto. Linfoma de Burkitt. Enfermedad de Hodgkin celularidad mixta.

1. Se trata de un defecto cualitativo de la cadena alfa de globina. 2. Precisa de vacunación antineumocócica. 3. Puede beneciarse de tratamiento con hidroxiu rea. 4. En el tratamiento no se realiza esplenectomía. 5. Hay predisposición a sepsis por Salmonella.

Un paciente consulta por debilidad y aparición de petequias en miembros inferiores. Entre sus valores hematológicos se nos citan las cifras de hematíes (2.500.000/mm3  y plaquetas (20.000/mm3. El diagnóstico MENOS probable sería:

66.

1. Leucemia aguda mieloblástica. 2. Leucemia aguda linfoblástica. 3. Coagulopatía de consumo.

4. Síndrome de Evans. 5. Ferropenia.

63.

Varón de 55 años que consulta por debilidad y molestias en hipocondrio izquierdo. En la exploración física se palpa esplenomegalia llamativa y el paciente presenta palidez mucocutánea. El hemograma es el siguiente: leucocitos 3.500 /microL con 80% de linfocitos atípicos, hemoglobina 7 g/dL, VCM 85 fL, plaquetas 50.000 /microL. Los linfocitos presentan positividad para marcadores CD20 y CD25. Respecto al diagnóstico más probable del paciente es falso:

Paciente de 35 años en tratamiento anticomicial 1. El aspirado de médula ósea frecuentemente es hi pocelular.

crónico por crisis de gran mal epiléptico. Reere

astenia y debilidad de varios meses de evolución. El hemograma es el siguiente: leucocitos 5.5 109/L con fórmula normal, hemoglobina 9 g/dL, VCM 110, plaquetas 140 x 10 9/L, reticulocitos 25 x 109/L. En el frotis sanguíneo se visualizan granulocitos con núcleos hipersegmentados. ¿Cuál es la causa más probable entre las siguientes?:

2. Se acompaña de inmunodeciencia celular.

3. El curso evolutivo es habitualmente lento. 4. El alotrasplante de progenitores hematopoyético es necesario en la mayoría de los pacientes. 5. Ocasionalmente se asocia a vasculitis.

67.

1. Anemia perniciosa. 2. 3. 4. 5.

64.

Niño de 8 años, de raza negra, con antecedentes familiares de anemia y crisis de anemia grave relacionadas con infecciones. Ha ingresado por episodios de insuciencia respiratoria aguda no infec ciosos y dolores óseos. Respecto al diagnóstico más probable del paciente, no es cierto:

Deciencia de folato. Hepatopatía por fármacos anticomiciales. Deciencia nutricional de vitamina B12. Síndrome mielodisplásco.

Las tienopiridinas son fármacos antiagregantes plaquetarios que actúan inhibiendo de forma irreversible el receptor plaquetario:

Mujer de 68 años que consulta por astenia y pérdida reciente de peso. En la exploración física se palpan poliadenopatías y esplenomegalia. La biopsia de una adenopatía demuestra sustitución de la arquitectura ganglionar normal por grandes nódulos con células pequeñas de núcleo hendido y grandes de núcleo redondo positivas para marcador CD20. En referencia al proceso de la paciente es falso: 1. Se trata de un linfoma indolente. 2. A menudo se encuentra extendido en el momento del diagnóstico. 3. La expresión en sangre periférica es excepcional. 4. En estadios avanzados es improbable la curación de la mayoría de los enfermos.

1. GPIb. 2. P2Y12. 3. GPIIb/IIIa.

-12-

EXAMEN ETMR 19/12

2. Alotrasplante de progenitores hematopoyéticos.

5. Está ligado al oncogén BCL-1.

3. Poliquimioterapia CHOP.

4. Fludarabina con ciclofosfamida y rituximab. 5. Cladribina.

68.

Mujer de 20 años, asintomática, que consulta tras realizarse un estudio que aporta los siguientes resultados: leucocitos 6.500 /microL, con fórmula normal, hematíes 5.100.000 /microL, hemoglobina 11 g/dL, VCM 67 fL, plaquetas 180.000 /microL. Sideremia 150 mg/dL, concentración de transferrina 350 mg/dL, ferritina 180 ng/mL. Estudio de hemoglobinas: A2 2%, F 0.3 %. ¿Cuál es el diagnóstico más probable?: 1. 2. 3. 4.

72.

Anemia de enfermedad crónica. Ferropenia en tratamiento. Rasgo talasémico beta. Rasgo talasémico alfa.

Varón de 35 años, sin antecedentes de interés, que acude a urgencias por dolor abdominal agudo. En la exploración se aprecia esplenomegalia y ausencia de ruidos intestinales, sin defensa. Rx de abdomen: dilatación de asas de intestino delgado sin nivel obstructivo. Angio-TAC: trombosis venosa mesentérica. ¿Qué estudio es el que aportará más información acerca del diagnóstico más probable del paciente?: 1. Niveles y actividad de antitrombina.

5. Anemia sideroblástica.

2. Niveles y actividad de proteína C y proteína S.

3. Estudio de mutaciones de JAK2. 4. Citometría de ujo sanguínea para reconocer cé lulas decientes en CD59 y CD55.

69.

Entre los estados de trombolia hereditaria se en-

5. Estudio de mutaciones del gen de protrombina.

cuentran todos los siguientes, con excepción de: 73.

1. Hemoglobinuria paroxística nocturna. 2. Protrombina 20210. 3. Deciencia de antitrombina. 4. Deciencia de proteína C. 5. Aumento de inhibidores de brinolisis.

El síndrome POEMS es una variante infrecuente de mieloma múltiple, donde es característico que aparezcan las siguientes manifestaciones excepto: 1. Lesiones cutáneas.

70.

2. 3. 4. 5.

El tratamiento de la enfermedad venooclusiva hepática post-trasplante de progenitores hematopoyéticos es: 74.

1. Ciclosporina.

2. 3. 4. 5.

Esteroides. Heparina. Fibrinolíticos. Tacrolimus.

Alteraciones endocrinológicas. Polineuropatía. Hepatomegalia. Afectación pulmonar.

¿Cuál de las siguientes es la enfermedad más probable en un niño de 7 años que presenta una hemoglobina de 7g/dL con VCM 70 fL y un índice de protrombina del 50%?: 1. Leucemia aguda mieloblástica. 2. Leucemia aguda linfoblástica.

71.

3. Enfermedad celíaca.

Enfermo de 52 años, sin antecedentes médicos de interés, que consulta por astenia. En la exploración física se palpan poliadenopatías y esplenomegalia. El hemograma es el siguiente: leucocitos 40.000 /microL con 90% linfocitos pequeños, hemoglobina 10 g/dL, plaquetas 215.000 /microL. El inmunofenotipo de los linfocitos es IgM-lambda, CD5, CD19, CD20. En el estudio citogenético las

4. Talasemia major.

5. Aplasia de médula ósea.

75.

células presentan del (13q). El tratamiento más

adecuado es: 1. Clorambucilo.

-13-

Un paciente de 50 años acude a Urgencias con un intenso dolor en el ojo izquierdo, que le sobrevino de forma brusca mientras dormía. En la exploración, el ojo afecto está enrojecido, con discreto edema corneal, y la pupila se encuentra en midriasis media, con pobre respuesta a la luz. Teniendo en cuenta el diagnóstico más probable, ¿qué opción

EXAMEN ETMR 19/12

3. Hemovitreo. 4. Endoftalmitis. 5. Membrana neovascular subretiniana.

le parece MENOS adecuada, entre las siguientes propuestas? 1. Si el paciente es hipermétrope, probablemente haya contribuido como factor predisponente. 2. Podría ayudarle el uso de manitol o acetazolamida. 3. Sería aconsejable el uso de midriáticos para rom per el bloqueo pupilar. 4. Los corticoides tópicos serían de utilidad para dis-

79.

que tenía pautadas. Acude a urgencias reriendo

pérdida brusca de visión en su ojo derecho, no acompañada de dolor ni de enrojecimiento ocular.

minuir el edema y la reacción inamatoria. 5. Es probable que este paciente reera ver halos al -

En la exploración inicial se pone de maniesto la

rededor de las luces.

76.

Paciente diabético tipo 2 de 20 años de evolución con muy mal control glucémico. En los últimos años no ha acudido a las revisiones oftalmológicas

ausencia de fulgor pupilar. Ante este cuadro, la primera posibilidad diagnóstica es:

Paciente con inyección ciliar, fotofobia, miosis y dolor ocular. El diagnóstico más probable es:

1. 2. 3. 4. 5.

1. Conjuntivitis bacteriana.

Obstrucción de arteria central de la retina. Obstrucción de vena central de la retina. Edema macular diabético. Hemovítreo. Neuropatía óptica isquémica anterior.

2. Iridociclitis. 3. Glaucoma agudo. 4. Desprendimiento de retina.

80.

5. Catarata.

77.

Ante un paciente con metamorfopsia, cuál de los siguientes diagnósticos considera más probable:

Un varón alcohólico indigente es remitido al Servicio de Urgencias en coma. En la analítica obtenemos los siguientes datos: pH 7,1, pO2 99 mmHg, PCO2 25 mmHg, creatinina 2 mg/dl, urea 50 mg/ dl, sodio 140 mmol/l, potasio 4 mmol/l, bicarbonato 5 mmol/l, cloro 90 mmol/l, proteínas 6 g/dl, glucosa 112 mg/dl, osmolalidad 320 mosm/kg. Orina:

no se observan cuerpos cetónicos, sodio 20 mmol/l, potasio 20 mmol/l, pH 5,5 y glucosuria negativa. ¿Cuál es la causa más probable de la acidosis?

1. Degeneración macular asociada a la edad. 2. Glaucoma crónico. 3. Glaucoma agudo de ángulo estrecho.

4. Retinosis pigmentaria. 5. Hemovítreo.

1. Acidosis respiratoria. 2. Cetoacidosis. 3. Acidosis láctica.

4. Intoxicación por metanol.

78.

Luisa es una paciente de 67 años de edad, con antecedentes de artrosis, dislipemia y bromialgia. Desde hace unos meses ha notado una disminución de visión bilateral, que se ha acentuado mucho la última semana en ojo derecho, acompañándose de metamorfopsia. Al explorar el fondo de ojo hallamos drusas blandas y alteración del epitelio pigmentario en la mácula de ambos ojos, y una hemorragia subretiniana en la mácula de ojo derecho. Tras realizar OCT y AGF se diagnostica a Luisa de una degeneración macular asociada a la edad en ambos ojos, siendo la de ojo derecho una forma exudativa. Se aconseja tratamiento en dicho ojo con fármacos an-

5. Insuciencia renal.

81.

Indique la relación cronológica FALSA referente al ciclo cardíaco: 1. A la apertura de la válvula semilunar le sigue la

expulsión ventricular. 2. Tras la relajación ventricular isovolumétrica se  produce la apertura de la válvula auriculoventri -

cular.

ti-VEGF (ranibizumab) por vía intravítrea. ¿Cuál

3. Tras la apertura de la válvula semilunar, tiene lu -

de las siguientes no es una complicación relacionada con esta vía de administración?

gar una contracción ventricular isovolumétrica. 4. A continuación del llenado lento ventricular, se  produce la sístole auricular. 5. La expulsión ventricular va seguida del cierre de

1. Hiposfagma. 2. Desgarro de retina.

la válvula semilunar.

-14-

EXAMEN ETMR 19/12

82.

Respecto a los sistemas para medir los diferentes espacios y volúmenes, señalar la FALSA:

85.

1. El volumen corriente se mide por espirometría simple.

Un varón de 25 años de edad adicto a drogas por vía parenteral tiene edemas en miembros inferiores. En la ecografía renal hay aumento de la ecogenicidad cortical en ambos riñones. La nefropatía que seguramente padece este enfermo, ¿a qué NO es secundaria?

2. Un método para medir la CRF es el de dilución de

helio. 3. El volumen residual puede medirse por pletismografía. 4. El espacio muerto anatómico se mide por espirometría forzada. 5. El espacio muerto siológico se mide por el méto do de Bohr.

1. Lupus eritematoso sistémico. 2. A.D.V.P (adicto a drogas por vía parenteral). 3. Reujo vesicoureteral.

4. Nefropatía túbulointersticial. 5. Nefropatía de cambios mínimos.

86. 83.

Un neonato, tras un parto distócico, presenta dicultad respiratoria y signos de insuciencia cardíaca. En la analítica se encuentra una creatinina sérica elevada, y la tensión arterial es alta. Acerca del trastorno que sospecha, señale la respuesta INCORRECTA:

Paciente de 31 años que acude a urgencias porque siente MEG, y dolor difuso en ambas fosas renales. Reere que tras un partido de fútbol ayer tomo una

pastilla cuyo nombre no recuerda porque le dolían las rodillas. Comenta que ha estado orinando más que habitualmente. En la analítica destaca Cr 3.1 mg/dL, Urea 120 mg/dL, Na 139 mEq/L, K 4.9 mEq/L. Leucocitos 10.000 (N 75%, L 20%, E 5%).

En la orina presenta Na 80 y en el sedimento leucocitos 30-40/campo, sin bacteriuria. ¿Primera medida?

1. La herencia es autosómica recesiva y se asocia a un desarrollo anómalo de los conductos biliares intrahepáticos.

2. La palpación abdominal y la ecografía son muy importantes para el diagnóstico. 3. La hipertensión portal es la regla al nal de la in fancia. 4. La ecografía abdominal mostrará riñones agrandados con aumento de la ecogenicidad.

1. Rehidratar y esperar la evolución. 2. Forzar diuresis para intentar transformar en poliúrico el FRA. 3. Iniciar inmunosupresores. 4. Prednisona 1mg/kg/día. 5. Retirada del fármaco.

5. La insuciencia renal no será oligúrica, pues la

tendencia es hacia la pérdida hidrosalina, en lugar de la retención.

87. 84.

Mujer de 80 años sin antecedentes personales de interés. Tras cuadro infeccioso de vías respiratorias, reere progresivo malestar generalizado,

astenia, anorexia y artralgias. Dos meses después se añadieron orinas oscuras hasta presentarse en oligoanuria y con edemas en extremidades inferiores. La exploración física fue normal salvo palidez de mucosas y fetor urémico. TA 160/95. Analítica: Cr 12 mg/dl, BUN 150 mg/dl, Hgb 8 gr/dl, Hto 24%, VCM 92, plaquetas 430.000/mm3, leucocitos

Niño de 11 años con los siguientes antecedentes: a los 6 años, y en varios análisis, se detecta hematuria microscópica, con niveles IgA normales y normocalciuria. A la edad de 9 años persistía la hematuria en los análisis e incluso habían observado algún episodio recortado de hematuria macroscópica. Un año más tarde se detectó proteinuria moderada de 1250 mg/24 horas. En el momento de la consulta persisten las alteraciones en el sedimiento, pero la proteinuria es de rango nefrótico, con creatinina sérica de 1,3 mg/dl. Existen antecedentes familiares de nefropatía evolutiva con desarrollo de insuciencia renal y de miopía familiar

por “lenticonus”. ¿Cuál sería el diagnóstico más probable?

12.000 (N 75%). Sedimento de orina: Cilindros eritrocitarios con cambios dismórcos y cilindros hialinos. Proteinuria cuanticada 2gr /dia. ¿Qué

no sospecharía? 1. Enfermedad de Alport. 2. Glomerulonefritis mesangial (enfermedad de Ber ger).

1. GN rápidamente progresiva.

2. S. nefrítico. 3. S. nefrótico.

3. Enfermedad poliquística autosómica dominante. 4. Enfermedad de adelgazamiento de la membrana  basal. 5. Nefritis intersticial por hipersensibilidad.

4. Inamación glomerular activa. 5. Insuciencia renal establecida.

-15-

EXAMEN ETMR 19/12

88.

Atendemos a un paciente de 78 años, diabético desde hace 15 años, con TA 120/80 mmHg y proteinuria. Tratamiento de elección en este momento:

4. Cr 6. 5. Cr 4.

93.

1. ARA II dado que no está hipertenso. 2. IECAs.

3. Betabloqueantes. 4. Antagonistas del calcio. 5. Espironolactona.

89.

¿Cuál de las siguientes afectaciones renales no es típica del virus de la hepatitis B?

Un paciente con enfermedad renal crónica estadio 4 secundaria a nefropatía vascular en situación de estabilidad, con tratamiento con IECAS para su control tensional, acude a la revisión con la siguiente analítica: Urea 120 mg/dl, cr 3 mg/dl, Na 135 mEq/L, K 4 mEq/L, Ca 8,5 mg/dL, P 6 mg/dL, PTH 200 ng/l. ¿Qué le añadiría en primer lugar al tratamiento? 1. ARA2. 2. Tolvaptán. 3. Análogos de la vitamina D.

1. 2. 3. 4. 5.

GN membranosa. FRA prerrenal. GN mesangio-capilar. Panarteritis nodosa. GN cambios mínimos.

4. Eritropoyetina. 5. Quelantes del fósforo.

94. 90.

Paciente de 20 años, diagnosticado de síndrome nefrítico agudo hace 3 meses. Presenta en la actualidad: proteinuria de 5 g/día, microhematuria, C3 persistentemente bajo, HTA y una Cr sérica de 3 mg/dl. Los anticuerpos anti-DNA son negativos y no tiene clínica sistémica. ¿Cuál es el diagnóstico más probable? 1. 2. 3. 4. 5.

En la nefronoptisis, se he descrito mutación de varios genes, ¿Cuál es el que produce manifestaciones clínicas más precoces? 1. 2. 3. 4. 5.

Persistencia de la GN aguda. Lupus eritematoso diseminado. GN membranoproliferativa. GN membranosa. Hialinosis focal.

95.

NPHP1. NPHP2. NPHP3. NPHP4. NPHP5.

Teniendo en cuenta que la pierna se divide en cuatro compartimentos (compartimento lateral, compartimento anterior y compartimento profundo, que a su vez se divide en compartimento posterior supercial y compartimento posterior profundo), una de las siguientes armaciones NO es cierta en

relación a los músculos de la pierna: 91.

La glomerulonefritis más frecuente en el área mediterránea es: 1. 2. 3. 4. 5.

1. El extensor largo del primer dedo se encuentra en el compartimento anterior de la pierna. 2. El tibial anterior es un importante extensor del to billo. 3. El peroneo breve es el principal realizador de la eversión del pie.

GN membranoproliferativa. GN membranosa. GN IgA. GN de cambios mínimos. GN postestreptocócica.

4. El tríceps sural es el principal exor plantar del

 pie. 5. El músculo poplíteo tiene una importante acción exora de la rodilla.

92.

¿Qué cromosoma está alterado en la poliquistosis renal autosómica recesiva? 96. 1. Cr 16. 2. Cr 11. 3. Cr 9.

-16-

El denominado manguito de los rotadores está compuesto por todos, MENOS uno de los siguientes músculos:

EXAMEN ETMR 19/12

1. 2. 3. 4. 5.

Subescapular. Supraespinoso. Redondo menor. Supraescapular. Infraespinoso.

100.

El cuadro clínico caracterizado por la desintegración del pensamiento abstracto, deterioro de la memoria y alteraciones tales como afasias, apraxias y agnosias, es característico de: 1. El estado confusional. 2. La epilepsia.

97.

Respecto a las condiciones de las aguas potables de consumo público, es cierto que:

3. El coeciente intelectual bajo.

4. La pseudodemencia depresiva. 5. La demencia degenerativa primaria.

1. La presencia de nitritos siempre es un indicador de contaminación fecal. 2. La presencia de amonio indica contacto con materia orgánica en descomposición, ocurrido con

101.

¿Cuál de los siguientes pacientes consideras que tiene un MENOR riesgo de suicidio?:

mucha antelación a la determinación. 3. Los niveles guía no corresponden a la calidad deseable en el agua potable. 4. Las concentraciones máximas admisibles corres -

1. Varón, 75 años, recientemente se ha quedado viudo, tiene un cáncer de próstata avanzado en tratamiento

 ponden a la máxima calidad del agua potable.

 paliativo, se encuentra ligeramente desanimado.

5. Las aguas blandas son agresivas.

98.

2. Mujer, 26 años, recibe el alta hospitalaria tras un  primer episodio de esquizofrenia, ha mejorado de

los síntomas psicóticos pero se encuentra bastante angustiada. 3. Varón, 55 años, alcoholismo crónico, graves pro  blemas familiares derivados de su adicción, le han diagnosticado un hepatocarcinoma como evolu-

Con respecto a las vías de transmisión del VIH, una de las siguientes es FALSA:

ción de su cirrosis y está pendiente de decisión

terapéutica.

1. La vía sexual es más fácil desde el hombre a la mujer que de la mujer al hombre.

4. Mujer, 55 años, trastorno distímico, cuidadora sobre-

2. Son vías descritas la transmisión mediante sexo oral, vaginal y anal.

carga de su madre, afectada de demencia avanzada. 5. Varón, 63 años, enfermedad bipolar, ingresado en

un centro residencial ante los problemas sociales derivados de su enfermedad, en estudio por la sos-

3. La transmisión vía oral es más difícil que con el

sexo vaginal. 4. El coito anal tiene un riesgo especial por la fragilidad de la mucosa rectal. 5. La existencia de otras ETS coexistentes no au-

 pecha de un cáncer de esófago, se encuentra muy

 preocupado en espera del diagnóstico.

menta el riesgo de transmisión de VIH vía sexual.

102. 99.

De los siguientes antipsicóticos, ¿cuál tiene menos efectos sedantes?

Un paciente que toma 20 mg de uoxetina desde

hace varios años por un síndrome depresivo comienza a notar inquietud, taquicardia, fasciculaciones musculares, sudoración profusa, escalofríos y aumento de la frecuencia de deposiciones a los pocos días de comenzar con un fármaco nuevo; ¿la interacción con cuál de los siguientes medicamentos NO es probable que sea la causa de este problema?

1. Olanzapina. 2. Tioridacina. 3. Clorpromacina.

4. Haloperidol. 5. Clozapina.

103.

¿Cuál de las siguientes armaciones es correcta

respecto a las parasomnias en los niños?

1. Isoniacida. 2. Tramadol. 3. Rizatriptán.

4. Ondansetrón.

1. Las pesadillas son más frecuentes durante la se -

5. Ciproheptadina.

gunda mitad del sueño. 2. En los niños con terrores nocturnos, el umbral del despertar es elevado, por lo que no es habitual que se despierten durante el episodio.

-17-

EXAMEN ETMR 19/12

3. Los terrores nocturnos pueden producir gritos, taquicardia y sudoración, mientras que esto es infrecuente en las pesadillas. 4. Los terrores nocturnos aparecen en el estadio de sueño REM. 5. Las pesadillas se relacionan con el trastorno por

107.

1. El tratamiento de elección es la carbamazepina. 2. Son pacientes con un mínimo de 4 episodios al año, maníacos o depresivos. 3. Son pacientes con episodios maníacos prolongados y depresivos cortos. 4. La distribución por sexos es prácticamente equitativa. 5. Responde mal al tratamiento con sales de litio.

estrés postraumático, mientras que los terrores

nocturnos se han asociado al sonambulismo.

104.

Respecto al concepto de ciclador rápido, no es cierto que:

Tras un fuego de campamento en el que se han contado historias de terror, un grupo de “boy-scouts” avisa corriendo a su monitor por haber visto un enorme animal entre las tiendas; al acercarse con una luz descubren que no hay nada y que sólo se habían confundido con las sombras que proyectaba la hoguera; ¿cómo llamaría a este fenómeno?

108.

Una de las siguientes armaciones sobre la epide -

miología de la esquizofrenia es FALSA: 1. En mujeres tiende a debutar más tardíamente.

1. 2. 3. 4. 5.

Alucinosis. Pseudoalucinación. Ilusión catatímica. Paraeidolia. Representación.

2. La prevalencia-vida es cercana al 1%. 3. Un 5% de los pacientes fallecerán por suicidio. 4. La recuperación completa se produce más del

70% de los casos. 5. El pronóstico social es mejor en los países en vías

de desarrollo.

105.

En la exploración física de un paciente con esquizofrenia paranoide en tratamiento antipsicótico de mantenimiento se evidencia un peso de 110Kg para una altura de 1’70m, con un perímetro abdominal de 105 cm, y tensión arterial de 145/90; en los análisis se detecta glucemia en ayunas de 125 mg/dL, con un perl lipémico alterado (co lesterol total 250 mg/dL, triglicéridos 225 mg/ dL, HDL 35mg/dL). ¿Cuál de los siguientes psicofármacos es más probable que esté tomando el paciente? 1. 2. 3. 4. 5.

106.

109.

1. 2. 3. 4. 5.

Ziprasidona. Paliperidona. Olanzapina. Aripiprazol. Sertindol.

110.

En el trastorno amnésico de Korsakov se afecta de

forma fundamental: 1. 2. 3. 4. 5.

Uno de los siguientes cuadros se caracteriza por un cambio progresivo de la personalidad, generalmente combinando la apatía y los problemas en el cuidado personal, con comportamientos inadecuados y desinhibidos, para posteriormente progresar hasta una demencia global:

La memoria reciente. La memoria implícita. La memoria procedimental. La memoria inmediata. La memoria remota.

Enfermedad de Huntington. Enfermedad de Pick. Encefalopatía de Binswanger. Trastorno de Korsakov. Demencia con cuerpos de Lewy.

Un varón de 35 años fue diagnosticado a los 18 años de esquizofrenia paranoide, habiendo sufrido varios brotes psicóticos hasta la fecha. Desde hace un tiempo la clínica del paciente se caracteriza por una gran pobreza del lenguaje, aislamiento social marcado, anhedonia y aplanamiento afectivo. Ya no presenta delirios ni alucinaciones auditivas claras. Usted escogería para el tratamiento de este paciente: 1. Haloperidol. 2. Clorpromacina.

3. Risperidona. 4. TEC.

-18-

EXAMEN ETMR 19/12

5. No existe ningún tratamiento que al menos mitigue parcialmente para los síntomas negativos de la esquizofrenia.

111.

3. El aislamiento debe ser las dos primeras semanas de enfermedad y no más allá tras la primera de

ictericia. 4. Es especialmente importante el lavado de manos. 5. La viremia es sólo transitoria.

Un paciente de 40 años, sin antecedentes de interés, consulta por disfagia. Ésta no se produce al iniciar la deglución, sino unos segundos después. El cuadro ha evolucionado progresivamente durante los últimos tres años, y actualmente es constante. Ocasionalmente, padece episodios de dolor retroesternal, de carácter opresivo. Ha perdido

114.

una escala de Glasgow de 8, hipotenso, disneico

y con heridas en ambas extremidades inferiores compatibles con fracturas abiertas: en la pierna derecha presentaba una herida puntiforme en la cara anterior de la pierna, por la que asoma un

peso, aunque no de forma signicativa, desde el comienzo del cuadro (4 kg en 3 años). Reere la

reciente aparición de regurgitaciones no ácidas desde hace pocos meses. Señale el diagnóstico más probable:

fragmento óseo (que aparentemente corresponde

con la tibia); a nivel del muslo izquierdo presenta una herida de diez centímetros, que expone la masa muscular del cuádriceps y que según el 061 exponía una fractura abierta de fémur, que redu jeron mediante tracción e inmovilizaron con férula neumática para el traslado del paciente; a nivel de la pierna izquierda presente una herida de 3 cm, con erosiones circundando la herida, pero sin exposición de fragmentos óseos. La exploración vascular de los miembros inferiores es normal. La exploración neurológica de los miembros inferiores no es valorable por el grado de inconsciencia

1. Cáncer de esófago.

2. Estenosis péptica. 3. Espasmo esofágico difuso. 4. Cáncer de esófago. 5. Acalasia esofágica.

112.

Tras una paliza brutal en la puerta de una discoteca, un joven de 30 años llega al Servicio de Urgencias remitido por el 061. Está inconsciente, con

Un paciente de 61 años, antiguo bebedor importante, padece cirrosis e hipertensión portal desde

del paciente. Al realizar el estudio radiográco

simple, correlacionándolo con el cuadro clínico, llegamos al diagnóstico de fracturas abiertas múltiples en MMII. En la valoración inicial, usted debe realizar el llamado protocolo ABCDE de la atención al paciente politraumatizado. Señale la medida incorrecta en relación con las cinco letras que forman parte del mismo:

hace varios años. Reere un aumento progresivo

del perímetro abdominal durante las últimas dos semanas, aunque está en tratamiento con espironolactona. No dice haber aumentado el consumo de sal. No tiene ebre ni dolor abdominal. La ex ploración revela ascitis franca, aunque no a tensión. Señale la respuesta FALSA en relación con este caso clínico:

1. Control de la vía aérea y de la columna cervical. 1. Está indicada la realización de una paracentesis

2. Respiración y ventilación.

diagnóstica. 2. En estos pacientes, a mayor hipoalbuminemia, es

3. Valoración circulatoria, control de los puntos san-

grantes activos.

más probable la aparición de la ascitis.

4. Valoración de posibles lesiones neurológicas.

3. Un gradiente seroascítico de la albúmina menor de 1,1 g/dL orientaría a hipertensión portal.

5. Exploración genitourinaria.

4. Probablemente habrá que incrementar la dosis

de espironolactona, e incluso añadir furosemida, aunque ésta implica cierto riesgo de encefalopatía

115.

hepática.

5. En pacientes con sospecha de ascitis, el signo de la matidez cambiante apoya el diagnóstico.

113.

Mujer de 42 años con antecedente de depresión mayor y en tratamiento desde hace 3 días con uoxetina que acude a urgencias por intento auto lítico con ingesta de sosa cáustica hace 3 horas. En la exploración oral se observa lengua depapilada y negruzca. ¿Cuál será la primera exploración a realizar?:

En cuanto a las medidas preventivas en el VHA, señalar la FALSA: 1. Es más leve y breve la clínica en los niños.

1. Gastroscopia. 2. Rx tórax y abdomen. 3. Ecografía abdominal.

2. El aislamiento recomendado es el estricto.

4. TC toraco-abdominal.

-19-

EXAMEN ETMR 19/12

3. Realizar colonoscopia bianual, hasta alcanzar la edad que tiene el paciente afectado. 4. Realizar colonoscopia y repetirla cada 2 años si se encuentran pólipos, realizando colectomía si aparece displasia severa. 5. Realizar colonoscopia, si se encuentran pólipos  programar cirugía, a ser posible no antes de los 20 años.

5. Tránsito con gastrogran.

116.

Se encuentra usted nuevamente en el servicio de Urgencias. Está atendiendo a otro varón de 30 años. Tiene también historia de úlcera duodenal, pero últimamente no tenía ningún síntoma. Se encuentra muy cansado y débil, nota palpitaciones y cuando se levanta se marea. Usted comprueba la palidez cutánea evidente y una frecuencia cardíaca de 120 lpm, aunque está afebril. ¿Qué es lo que NO considera correcto con este enfermo?

120.

auricular crónica en tratamiento con digoxina y antiagregantes, que presenta bruscamente dolor, palidez, anestesia e impotencia funcional en pie, pierna y mitad inferior de muslo derecho. A la exploración hay ausencia de pulsos pedio, tibial posterior y poplíteo derecho. Señale el diagnóstico más probable:

1. No pensaría en complicaciones de la úlcera por no tener molestias pépticas. 2. La exploración física está incompleta. 3. Indagaría en el posible consumo de antiinamato -

rios. 4. Probablemente tenga cifras de urea por encima de lo normal con creatinina normal en sangre. 5. Probablemente esté indicada una endoscopia.

117.

1. Trombosis venosa profunda. 2. Trombosis venosa supercial.

3. Arteritis de Takayasu. 4. Embolia arterial aguda. 5. Enfermedad de Buerger.

Señale de entre las siguientes, cual es causa de diarrea motora: 121.

Varón de 55 años estudiado por astenia que presenta la siguiente analítica: Hb 14.2, VCM 78.5 , HCM 33 pg, leucocitos 7500 con 74% neutrólos,

1. Hidróxido de magnesio.

plaquetas 195.000, actividad protrombina 60%, urea 29 mg/dl, creatinina 0.6 mg/dl, GOT 1086, GPT 1098, GGT 1180, bilirrubina total 2.2 mg/dl, proteínas totales 6.8 g/dl, albúmina 4.5 g/dl, fosfatasa alcalina 68 U/L, LDH 168, colesterol 136 mg/ dl, triglicéridos 92 mg/dl, sodio 142, potasio 3.9. Serología: antiHBs > 100 UI/ml, AgHBs +, AntiHBc -, AntiHBc IgM +, AgHBe +, antiHBe-, DNA-VHB 98.000 UI/ml, anti-VHC -, RNA VHC -, AgVHD -, IgG CMV +, IgM CMV -, IgG Ebstein Barr +, IgM Ebstein Barr -, IgG-VHA +. Señale la opción más correcta acerca del diagnóstico del paciente.

2. Enfermedad de Crohn. 3. Colitis por Salmonella. 4. Vipoma.

5. Hipertiroidismo.

118.

Mujer de 50 años con antecedentes de brilación

El test de la D-xilosa alterado indica: 1. Defecto de la digestión de los hidratos de carbono. 2. Síndrome de malabsorción por defecto de la pared del intestino. 3. Esteatorrea pancreática. 4. Décit congénito de disacaridasas.

1. Hepatitis crónica VHC. 2. Hepatitis fulminante VHA. 3. Hepatitis crónica B por cepa salvaje.

5. Obstrucción biliar.

4. Hepatitis crónica B por cepa mutante de escape. 5. Mononucleosis infecciosa.

119.

Un paciente es diagnosticado de poliposis adenomatosa familiar. ¿Qué recomendaría a un hermano suyo de 18 años?

122.

Los pólipos benignos más frecuentes en el intestino son: 1. Hamartomas.

1. Screening mediante análisis del gen APC en leu-

2. Pólipos inamatorios.

cocitos de sangre periférica.

3. Lipomas.

2. Colonoscopia anual y colectomía, si se detectan

4. Pólipos hiperplásicos.

 pólipos.

5. Adenomas.

-20-

EXAMEN ETMR 19/12

123.

Mario es un paciente varón, que acude al servicio de urgencias por un cuadro de 6 días de evolución de ictericia cutánea y conjuntival. Al realizar

1. El daño aparente (lesión cutánea) suele ser más

aparatoso de lo que en realidad son las lesiones internas. 2. La resistencia a la corriente eléctrica es muy esca-

la anamnesis nos reere una pérdida de peso de

15 Kg en los últimos 6 meses. Entre las pruebas diagnósticas solicitadas está un TAC, que pone

sa en el tejido nervioso y músculos, y elevada en la piel, salvo cuando está mojada. 3. Si aparece daño renal es improbable que se justi que por rabdomiolisis, ya que la afectación mus -

de maniesto una masa en los conductos biliares

intrahepáticos, motivo por el que se realiza una colangiografía, que nos proporciona el diagnóstico de colangiocarcinoma. Señale la respuesta INCORRECTA sobre el colangiocarcinoma:

cular es rara. 4. La luxación anterior de hombro es la que típicamente se asocia a descargas eléctricas y crisis convulsivas. 5. Si se producen a nivel doméstico se hace muy im probable la aparición de arritmias cardíacas.

1. Se relaciona con enfermedades que producen colestasis crónica, como la enfermedad de Caroli,

126.

los quistes de colédoco o la colangitis esclerosante primaria. 2. Se relaciona con el virus de la hepatitis B. 3. El tratamiento de elección es la resección hepáti ca, aunque pocas veces es posible. 4. La recurrencia en el trasplante es muy frecuente,  por lo que no es aconsejable realizarlo. 5. Es más frecuente en pacientes ancianos que a eda des jóvenes.

124.

Un paciente intervenido por ulcus duodenal con una antrectomía y gastroyeyunostomía tipo Billroth II presenta una recaída ulcerosa. La gastrina sérica está elevada basalmente y aumenta de forma importante tras inyección de secretina. ¿Qué sospecharía? 1. Adenocarcinoma gástrico.

2. 3. 4. 5.

Mujer de 25 años que acude al hospital por mal estado general, náuseas con vómitos alimentarios e ictericia de 1 semana de evolución. A la exploración destaca una ictericia cutáneo-mucosa intensa, ligera ascitis sin hepatomegalia y confusión. En la analítica presenta GOT 3.560 U/ml, GPT 4.520 U/ml, bilirrubina 35 mg/dl con directa de 20 mg/ dl, leucocitosis con desviación izquierda, tiempo de protrombina del 20%. ¿Cuál es el diagnóstico MENOS probable?

127.

Infección por H. pylori. Antro retenido. Gastrinoma. Síndrome de asa aferente.

Un paciente con antecedentes de etilismo crónico ingresa en el hospital por astenia y anemia intensas. En la exploración física se objetiva hepatoesplenomegalia y en los estudios sanguíneos, acantocitos e hiperlipemia. El cuadro es característico de: 1. Hemólisis por hiperesplenismo. 2. Enfermedad de Wilson. 3. Síndrome de Zieve.

1. Hepatitis aguda fulminante - VHA.

2. Hepatitis aguda fulminante - intoxicación por setas. 3. Hepatitis aguda alcohólica. 4. Hepatitis aguda fulminante - autoinmune.

4. Deciencia de folato.

5. Saturnismo.

5. Hepatitis aguda fulminante - VHC.

128. 125.

Trasladan a nuestro centro un paciente al que han encontrado caído e inconsciente junto a una torre de alta tensión. Al ingreso, el paciente ha recupe-

Varón de 58 años diagnosticado de cirrosis por VHC que acude a urgencias por deterioro del nivel de conciencia de 5 horas de evolución. La exploración física mostraba hepatomegalia leve y matidez

rado el conocimiento, presentando un Glasgow de

cambiante en ancos. La analítica en urgencias

15 puntos, y se constata la existencia de inestabilidad hemodinámica. A nivel del tórax presenta una quemadura de segundo grado. En el brazo derecho presenta una quemadura con carbonización de la mano, necrosis tisular extensa y fractura abierta de cúbito y radio. En el brazo izquierdo el paciente presenta dolor e impotencia funcional, con dolor a la movilización pasiva de dicho miembro superior. En ambos muslos presenta quemaduras de primer

fue la siguiente: Hb 13.2 g/dl, plaquetas 80.000/ mm3, leucocitos 9.000, glucosa 88 mg/dl, urea 60 mg/dl, creatinina 1,5 mg/dl, bilirrubina total 2.4 mg/dl, GOT 63 U/l, GPT 72 U/l, GGT 81 U/l, fosfatasa alcalina 154 U/l, amilasa 29 U/l, LDH 119 U/l, PCR 3.0 mg/l, sodio 128, potasio 4.3. ¿Cuál de entre las siguientes NO sería una opción aconsejada?

grado profundo. Entre las siguientes armaciones

1. 2. 3. 4.

sobre las quemaduras eléctricas, señale la CORRECTA:

-21-

Sueroterapia. Albúmina i.v. Restricción hídrica. Paracentesis diagnóstica.

EXAMEN ETMR 19/12

5. Enemas de lactulosa.

132.

Un enfermo de prótesis valvular cardíaca va a ser sometido a una exploración mediante colonoscopia. ¿Cuál de las siguientes le parece la pauta de prolaxis más adecuada?

129.

Varón de 47 años, fumador de 15 cig/día y bebedor activo de 60 gramos/día que acude a consulta por astenia. Realiza una analítica donde se observa: Hb 13.8 g/dl, leucocitos 4500/mm3  con 70% neutrólos, plaquetas 88.000/mm3, glucosa 90 mg/dl, urea 38 mg/dl, creatinina 1.1 mg/dl, GOT 105 U/L, GPT 76 U/L, GGT 200 U/L, bilirrubina total 1.8 mg/dl, proteínas totales 5.0 g/dl, albúmina 2.0 g/dl, fosfatasa alcalina 101 U/L, LDH 220, sodio 140, potasio 3.9. Realiza una ecografía donde observa una hepatomegalia leve con un hígado heterogéneo de predominio hiperecogénico con bordes irregulares, sin otros hallazgos de interés. La gastroscopia observó una mucosa parcheada astral. Se tomo biopsia, siendo informada como gastritis crónica super cial. ¿Cuál sería la actitud más adecuada?:

1. 2. 3. 4.

5. Clindamicina intravenosa.

133.

¿Qué actitud tomaría ante un varón de 65 años, asintomático y con buen estado general, que presenta en la placa de tórax residuos brosos sugestivos de tuberculosis antigua (sin presentar signos

de tuberculosis activa) y que nunca ha recibido tratamiento antituberculoso?

1. Iniciar tratamiento espironolactona 100 mg/día. 2. Repetir endoscopia oral en 1-2 años.

1. Abstención terapéutica. 2. Observación y revisión a los 6-12 meses.

3. Trasplante hepático.

3. Quimioprolaxis con isoniacida 300 mg/día 12

4. Iniciar propranolol.

meses. 4. Tratamiento con isoniacida 300 mg/día y rifampicina 600 mg/día durante 9-12 meses. 5. Estudio de resistencias del  M. tuberculosis, eligiendo el tratamiento en función de éste.

5. Noroxacino oral.

130.

Amoxicilina vía oral. Ampicilina intravenosa. Eritromicina vía oral. Ampicilina y gentamicina intravenosas.

Marca la respuesta correcta: en el esófago de Barrett, la realización de una técnica antirreujo,

con demostración manométrica y pHmétrica, de 134.

ser ecaz:

1. Hace retroceder siempre las alteraciones de la mucosa esofágica.

2. No hace retroceder la enfermedad, pero impide la evolución a carcinoma. 3. No desaparece el potencial para que se desarrolle un adenocarcinoma. 4. Nos exime de realizar seguimientos endoscópi-

superior derecho, lo que se conrma en el estudio radiográco con la presencia de un inltrado en

cos, siendo suciente el seguimiento clínico.

buena parte del lóbulo superior del pulmón derecho, junto con pequeñas lesiones redondeadas múltiples en lóbulo superior de ambos pulmones, que se comprueban están en relación con su proceso previo de silicosis. La saturación de oxígeno es del 96% sin administrar O2. Se solicita una analítica en la que se obtiene: leucocitos 14.500/ml; hemoglobina 15,4 g/dL; hematocrito 45%. Se realiza test de Mantoux y toma de esputo para baciloscopia y cultivo. El Mantoux a las 48 horas resulta

5. Tras dos estudios consecutivos de pHmetría esofágica normal, separados entre sí por al menos 12

meses, puede prescindirse de nuevas valoraciones del paciente.

131.

Paciente de 66 años de edad, ex-minero con antecedentes de silicosis desde hace 25 años, que acude a urgencias por proceso febril de hasta 39ºC, de unos 3 días de evolución, acompañado de tos, que en el día de hoy se ha acompañado de expectoración hemática. El paciente reere que en los últi mos 4 meses ha perdido 6 o 7 Kg de peso, “sin hacer dieta, ni nada especial”. A la exploración física al paciente le auscultamos estertores en el lóbulo

¿Cuál de las siguientes NO es una complicación tardía de las luxaciones traumáticas de cadera?

positivo (15 mm), la tinción para BAAR resulta 1. Artritis postraumática.

negativa, pero el cultivo, tanto en BACTEC como

2. Necrosis aséptica de la cabeza femoral. 3. Miositis osicante.

en medio de Löwenstein-Jensen, resulta positivo para  Mycobacterium tuberculosis . Respecto a este

4. Inestabilidad con luxación recidivante.

germen señale la respuesta falsa:

5. Neuroapraxia del nervio ciático.

-22-

EXAMEN ETMR 19/12

1. Es un bacilo ácido alcohol resistente aerobio es-

3. En principio, un tratamiento correcto sería cloxa-

tricto.

cilina + gentamicina. 4. La válvula afectada con mayor frecuencia es la

2. Su estructura presenta gran cantidad de ácidos mi-

cólicos. 3. Tiene un factor de virulencia denominado “cordfactor”. 4. Es no esporulado, inmóvil y no productor de toxinas. 5. El tiempo de replicación es de 10 a 12 días.

mitral. 5. El cuadro podría complicarse con una insucien -

cia tricuspídea grave.

138. 135.

Un paciente consulta por ebre, tos y dolor torá -

cico, después de volver de unas vacaciones en un lujoso hotel, donde veraneaba en la playa. Tam-

Paciente varón de 35 años que consulta por un cuadro caracterizado por presencia de disuria y secreción uretral de tipo purulento, que se inicia tres días después del contacto sexual. Indique cuál es el agente etiológico más probable:

bién reere diarrea. Se objetiva una Tª de 38,8 ºC.

Desde el punto de vista analítico, destaca una leucocitosis con desviación izquierda, así como unas

1.  Neisseria gonorreae. 2. Trichomona. 3. Chlamydia trachomatis .

cifras de Na+ de 131. Rx tórax: inltrados difusos

bibasales, de carácter alveolar. En el sedimento de orina, se detectan 8-10 hematíes por campo. Se conrma el diagnóstico mediante un test de detección de antígeno en orina. ¿Cuál de los siguientes antibióticos es de elección en la actualidad para el tratamiento de esta infección?

4. Virus Herpes. 5. Candida spp.

139.

Un paciente de 71 años, alcohólico, en tratamiento con prednisona en dosis decreciente por una hepatitis aguda alcohólica (actualmente 30 mg diarios),

consulta en el Servicio de Urgencias con un cuadro de tres días de evolución de cefalea, náuseas, vómitos, sensación distérmica y malestar general. El paciente está febril, somnoliento y con rigidez de nuca, sin otros hallazgos. Se realiza una TC cerebral, que descarta lesiones ocupantes de espacio, seguido de una punción lumbar, donde se objetiva hipoglucorraquia, hiperproteinorraquia y linfocitos en LCR. En espera de los resultados del cultivo, el tratamiento empírico más correcto sería:

1. Doxiclina. 2. Eritromicina. 3. Levooxacino.

4. Rifampicina. 5. Ceftriaxona.

136.

Sobre la pleuresía tuberculosa, señale lo FALSO: 1. 2. 3. 4.

El comienzo suele ser brusco. El derrame es un exudado. Hay muchas células mesoteliales. La cutirreacción a la tuberculina es negativa en una tercera parte de enfermos. 5. La respuesta a la quimioterapia es buena.

137.

1. 2. 3. 4.

Ceftriaxona. Ceftriaxona y vancomicina. Ceftriaxona, vancomicina y ampicilina. Cefotaxima y vancomicina.

5. Aciclovir.

Paciente de 30 años de edad, adicto a drogas por vía parenteral, que acude al servicio de urgencias por ebre de 39,5 ºC, dolor pleurítico, tos y expectoración. En la radiografía de tórax se observan lesiones nodulares periféricas múltiples, algunas de ellas cavitadas. Se realiza un ecocardiograma,

140.

Varón de 52 años que consulta por cuadro de 2 meses de evolución consistente en astenia, febrícula vespertina, pérdida de 5 kg de peso y tos seca,

que en los dos últimos días se ha acompañado de esputos hemoptoicos. No es fumador y trabaja en el campo. Aporta radiografía de tórax realizada hace 24 horas con patrón destructivo biapical. La exploración física es normal. ¿Qué prueba recomendaría en este momento?

que conrma el diagnóstico de sospecha. ¿Cuál sería, de entre las siguientes, la armación INCO-

RRECTA? 1. El microorganismo más probable es S. aureus.

1. Auramina de esputo. 2. Anticuerpos monoclonales para  Pneumocystis jirovecii en esputo.

2. Es posible la ausencia de soplos en la auscultación cardíaca.

-23-

EXAMEN ETMR 19/12

3. Fibrobroncoscopia. 4. Punción transtraqueal.

1. Tetraciclinas. 2. Ciprooxacino. 3. Amoxicilina-Clavulánico. 4. Ceftriaxona.

5. Inmunouorescencia directa para  Legionella  pneumophila en esputo.

5. Azitromicina.

141.

Un varón de 30 años, fumador habitual de medio paquete al día, pescadero de profesión y cazador de palomas torcaces en sus ratos libres, consulta por presentar una mácula eritematosa, caliente y dolorosa en el brazo derecho. A la exploración física destaca además la existencia de febrícula, de lesiones vesiculosas sobre el área de eritema, y de una adenopatía móvil en la axila derecha. Tras

144.

1. 2. 3. 4. 5.

una anamnesis detallada el paciente reere que

mantiene relaciones sexuales habitualmente no protegidas, que convive con un perro y dos gatos, y que dos días antes se pinchó con una espina de atún en el dedo índice de la mano derecha. Señale cuál es el agente más probable de este cuadro, y qué tratamiento considera indicado: 1. 2. 3. 4. 5.

142.

Un niño que ha sido sometido a esplenectomía NO tiene mayor riesgo de sufrir una infección por:

145.

Streptococcus pneumoniae .  Aspergillus fumigatus.  Haemophilus inuenzae .  Neisseria meningitidis.  Babesia microti.

Un pastor de ovejas de 23 años ingresa por un cuadro de insuciencia respiratoria con inltrados

pulmonares bilaterales, que precisa ventilación mecánica. Avisan de microbiología que en los hemocultivos se obtiene crecimiento bacilos grampositivos pendientes de identicar. El agente etiológico más probable es:

 Francisella tularensis. Estreptomicina. Sporothrix schenckii . Itraconazol.  Bacillus anthracis. Penicilina G. Staphylococcus aureus. Cloxacilina.  Erysipelothrix rhusiopathiae. Penicilina G.

1. 2. 3. 4. 5.

En relación a las infecciones del Sistema Nervioso por Virus Herpes Simple, señale la respuesta correcta:

 Legionella pneumophila. Coxiella burnetii .  Francisella tularensis.  Bacillus anthracis. Clostridium septicum.

1. La Encefalitis tiene predominio estacional, siendo más frecuente en invierno. 2. La etiología más frecuente de la meningitis linfo citaria benigna recurrente (meningitis de Molla ret) es el Virus Herpes Simple 1. 3. El cuadro clínico de Meningitis por Virus Herpes se debe con más frecuencia al Virus Herpes 1.

146.

Paciente de 27 años que reere disuria, leucorrea

mucopurulenta hemorrágica, ¿qué sospecha usted?

4. La Polirradiculopatía Lumbosacra en pacientes infectados por VIH, se debe a virus Herpes Sim  ple 2. 5. El diagnostico de la Encefalitis por Herpes Simple se realiza mediante su detección en L íquido Cefa -

1. Infección por Gardnerella. 2. Sílis.

3. Gonorrea. 4. Infección por C. trachomatis. 5. Infección por T. vaginalis.

lorraquídeo por técnica de PCR.

147. 143.

Una mujer de 42 años acude en el mes de julio a Urgencias por presentar desde hace 2 días ebre

de 40 ºC, cefalea holocraneal, mialgias y poliartralgias. Entre sus antecedentes sólo destaca que tiene un perro en su domicilio. Usted, tras recoger la historia de la paciente, ya sospecha qué enfermedad padece, por lo que enfoca su exploración

Mujer de 25 años con nódulo mamario palpable de aparición brusca. La ecografía revela un nódulo anecogénico, de límites muy precisos, morfología regular y refuerzo posterior, único, de 3 cm de diámetro. ¿Cuál de los que se relacionan es el diagnóstico más probable?

física al hallazgo de cierta lesión. Tras conrmar

1. Quiste.

su sospecha, le instaurará tratamiento con:

2. Displasia brosa.

3. Fibroadenoma.

-24-

EXAMEN ETMR 19/12

5. Administración de glucosa a la madre.

4. Cáncer.

5. Hamartoma.

151. 148.

Mujer de 47 años que acude a consulta de ginecología remitida desde su médico de atención primaria. Reere menstruaciones en cantidad mayor

a lo habitual en los últimos meses. Su médico le realizó analítica básica con hemograma, donde presentaba hemoglobina de 10 g/dL, por lo que le pautó tratamiento con hierro oral y decidió remitirla al ginecólogo. Para ampliar el estudio, se decide solicitar, entre otros, un estudio hormonal. Señale cuál es el resultado que esperaría encontrar con mayor probabilidad en esa analítica:

1. 2. 3. 4. 5.

152.

1. Disminución de la FSH y LH, con niveles normales o elevados de estradiol. 2. Aumento de LH, con FSH normal y leve disminución de estradiol. 3. GnRH y estrógenos disminuidos, con FSH y LH normal. 4. Elevación de FSH, con GnRH y estrógenos normales y LH normal o levemente aumentada. 5. Disminución notable de estradiol, con FSH, LH y GnRH normales.

149.

Ia. Ib. Ic. IIa. IIb.

Mujer de 35 años de edad, gestante de 36 semanas de edad gestacional, sin antecedentes personales de interés, que acude para realizar la prueba de monitorización no estresante (NST). En el mismo

no se aprecia ninguna aceleración de la FCF tras 20 minutos de prueba, y tras estimular manualmente al feto, tampoco hay aceleración alguna tras 20 minutos, considerando la prueba no estresante no reactiva. Por este motivo se decide realizar la prueba estresante o de Pose. Una prueba de POSE o de la oxitocina se considera positiva cuando:

En una mujer embarazada son correctos todas las

1. Aparecen desaceleraciones tardías o variables graves en menos del 30% de las contracciones. 2. Ante cualquier contracción aparece una desaceleración. 3. Aparición de desaceleraciones precoces con todas las contracciones. 4. Aparición de desaceleraciones tardías en el 30% de las contracciones o desaceleraciones variables graves con todas las contracciones. 5. Aparición de alteraciones de la FCF como res  puesta a la hipertonía uterina.

modicaciones gravídicas excepto una, señálela:

1. La frecuencia respiratoria aumenta. 2. La TA aumenta durante el tercer trimestre. 3. El eje eléctrico cardiaco se desplaza a la izquier -

da. 4. La frecuencia cardiaca disminuye. 5. El ltrado glomerular aumenta.

150.

¿En qué estadio se encuentra un carcinoma que afecta únicamente a los dos ovarios y en el que existe una ascitis de 400 cc citológicamente positiva?

Secundípara en la 39ª semana de gestación que

comienza con dinámica de parto. Entre sus antecedentes obstétricos destaca una cesárea anterior por una desproporción pelvicocefálica. El embarazo actual ha transcurrido con normalidad. Se

153.

hace un registro cardiotocográco que demuestra

una frecuencia cardíaca de 110 lpm. La amnioscopia nos muestra un líquido teñido de meconio. Se realiza una microtoma de sangre fetal, con un resultado de un pH de 7,24. ¿Cuál de las siguientes actuaciones es la MENOS útil?

Gestante a término que acude al Hospital en trabajo de parto. A la exploración, por tacto, se aprecia cuello borrado y centrado, dilatación de 4 cm, presentación cefálica en plano I. Tras una monitorización de 15 min, en la que se aprecia buen estado fetal y dinámica regular a intervalos de 4 min, se practica amniorrexis articial con obtención de

líquido profusamente teñido de sangre, apareciendo graves alteraciones del registro fetal. ¿En qué deberá pensar?

1. pH materno para descartar acidosis materna.

1. 2. 3. 4.

2. Colocar a la madre en decúbito lateral.

3. Repetir la microtoma en unos 15 minutos. 4. Suspender gotero oxitócico, si se estaba utilizando.

-25-

Placenta previa. Abruptio placentae. Rotura uterina. Rotura de vasos previos.

EXAMEN ETMR 19/12

5. Traumatismo del canal del parto.

154.

157.

¿Cuál de las siguientes opciones acerca de la mama y la lactancia materna es FALSA?

Gestante que presenta amenorrea de 10 semanas; ¿cuándo realizaría la determinación de alfafetoproteína? 1. Ya se ha pasado la fecha de realizar la determinación. 2. En la semana 16. 3. En la semana 20. 4. A partir de la semana 36. 5. En la semana 5.

1. Los alveolos desembocan en los conductos galactóforos, que se unen para formar los 12 a 15 conductos que desembocan en el pezón. 2. Las hormonas que participan en el desarrollo lo buloalveolar son: progesterona, estrógenos, esteroides suprarrenales, prolactina, hormona de crecimiento y lactógeno placentario durante el embarazo. 3. Los estrógenos bloquean la acción de la prolactina

158.

El síndrome de Transfusión Feto-Fetal se considera complicación típica de los embarazos gemelares:

en la mama, por eso no se produce eyección láctea

hasta que se expulsa la placenta y descienden sus niveles. 4. Durante los dos-tres primeros días de la lactancia se secreta una leche amarillenta, llamada calostro,

1. 2. 3. 4.

que se caracteriza por tener más grasa y menos

 proteínas y azúcares que la leche madura. 5. Entre los factores que contribuyen a mantener la lactancia, encontramos una correcta alimentación e ingesta hídrica, una buena succión y descarga del pecho en cada toma y factores psicológicos y sociales que actúan en la regulación de la secreción de prolactina y oxitocina.

155.

Monocigóticos bicoriales-biamnióticos. Bicigóticos bicoriales-biamnióticos. Bicigóticos monocoriales-biamnióticos. Obtenidos por técnicas de Reproducción Asistida.

5. Complicados con Preeclampsia materna.

159.

Cuál de las siguientes manifestaciones clínicas NO es característica de la preeclampsia:

Una gestante de 10 semanas que acude a recoger los resultados de su analítica de primer trimestre y en el cultivo de orina se encuentran >100.000 UFC de E. Coli. La paciente está asintomática. ¿Qué tratamiento debe recibir?: 1. Ninguno, dado que está asintomática.

2. 3. 4. 5.

1. Edema extracelular por lesión vascular e hipoproteinemia. 2. HTA por elevación de los niveles de renina y angiotensina. 3. Elevación de las transaminasas por necrosis hepatocelular. 4. Alteraciones occipitales por vasoespasmo. 5. Proteinuria por endoteliosis glomerular.

160.

Quinolonas. Fosfomicina. Estreptomicina. Tetraciclinas.

Decimos que una prueba diagnóstica es muy especíca cuando:

1. Aplicada a una población de enfermos, tiene pocos falsos negativos.

156.

Con respecto a la mastopatía broquística, señale

2. Cuando un resultado negativo es poco probable

la FALSA:

que corresponda a un enfermo. 3. Aplicada a una población de sanos, tiene pocos falsos positivos. 4. Tiene una alta sensibilidad diagnóstica. 5. Tiene pocos falsos positivos y pocos falsos negativos.

1. El síntoma más frecuente es la mastodinia.

2. El embarazo y la lactancia empeoran la mastopatía. 3. Es muy rara después de la menopausia. 4. Se deben hacer técnicas de imagen (ecografía y mamografía) y estudio histológico. 5. El riesgo de cáncer está moderadamente aumenta -

161.

do en los tipos proliferativos con atipia.

-26-

Para actuar correctamente sobre un determinado problema de salud, primero hay que conocerlo adecuadamente realizando, entre otros, la evalua-

EXAMEN ETMR 19/12

165.

ción del peso especíco y distribución del mismo

en la comunidad. Si quisiera realizar un estudio para conocer la prevalencia de dicho problema de salud elegiría un diseño de tipo: 1. 2. 3. 4. 5.

Estudio casos y controles. Estudio transversal. Ensayo clínico. Ensayo comunitario de intervención. Estudio ecológico.

Nos disponemos a estudiar los historiales médicos de los trabajadores de una fábrica desde hace 45 años. Entre los expuestos al producto químico “z” se observa una mayor incidencia a lo largo del tiempo de un tipo de cáncer. Estamos realizando un estudio: 1. De casos y controles. 2. Ensayo de campo. 3. Cohortes retrospectivo.

4. Ensayo clínico. 5. Descriptivo.

162.

Usted dispone de una prueba diagnóstica de sílis

y decide hacer un despistaje de la enfermedad en la población que atiende. Sabemos que la sensibi-

166.

lidad de la prueba es de 0,95 y la especicidad de 0,95. Si la prevalencia de la sílis en su población

es de 1/1.000 aproximadamente, ¿cuál será el valor predictivo positivo?

Respecto a los estudios de casos y controles, es FALSO que: 1. La medida de asociación que se emplea en este tipo de estudios es la odds ratio (OR).

1. 2. 3. 4. 5.

2. Aunque no permiten calcular incidencias, el valor de la odds ratio se aproxima al del riesgo relativo cuando se trata de enfermedades muy prevalentes. 3. Se analizan comparando la frecuencia de una ex posición en el grupo de casos respecto al grupo de controles. 4. Este diseño es especialmente vulnerable al llamado sesgo de memoria.

0,02 (2%). 0,95 (95%). 1,0 (100%). Con estos datos no se puede saber. 0,05 (5%).

5. Si seleccionáramos casos prevalentes en lugar de

163.

Le proponen examinar la precisión de un nuevo sistema de tiras reactivas para el análisis rápido de niveles plasmáticos de triglicéridos. Sobre una muestra patrón de plasma, realiza 200 mediciones, que arrojan una distribución de resultados cuya media es 150 mg/dl y cuya desviación típica es 5 mg/

casos incidentes de la enfermedad o proceso en estudio, podríamos incurrir en la falacia de Neyman.

167.

dl. ¿Cuál es el valor del coeciente de variación?

1. 150/5 = 30 mg/dl. 2. 5/150 = 0,033 mg/dl.

un servicio de neumología. Señale la armación

CORRECTA en relación con este estudio:

3. (5/150)x100 = 3,33 mg/dl. 4. (5/150)x100 = 3,33. 5. 5/V-200 = 0,35.

164.

1. Se aumenta el error aleatorio. 2. Existe un sesgo de selección. 3. El grupo control es adecuado, puesto que proviene del mismo servicio que los casos.

Actualmente en España se hace sistemáticamente a los recién nacidos una determinación de TSH para la detección precoz del hipotiroidismo. ¿Qué parámetro valorará la probabilidad de tener hipotiroidismo entre los que han resultado positivos a la prueba?

4. Sería mejor emplear un grupo control compuesto

 por no fumadores. 5. Existe un factor de confusión.

168. 1. Sensibilidad. 2. 3. 4. 5.

Se diseña un estudio de casos y controles para determinar si existe asociación entre el cáncer de pulmón y el hábito de fumar. Se seleccionan como controles un conjunto de pacientes ingresados en

Especicidad. Valor predictivo positivo. Valor predictivo negativo. Likelihood ratio (razón de probabilidades).

¿Qué tipo de estudio plantearía para investigar la relación entre un supuesto factor de riesgo y una enfermedad muy poco frecuente? 1. Estudio de cohortes.

-27-

EXAMEN ETMR 19/12

2. 3. 4. 5.

Estudio de casos-controles. Ensayo clínico. Estudio de corte. Ensayo de campo.

5. Serie de casos clínicos.

172.

En su servicio le han encargado diseñar un ensayo clínico para valorar la ecacia del tacrolimus tópico

169.

para mejorar los síntomas y signos en pacientes con queratoconjuntivitis sicca. Elija el tipo de diseño experimental que permitiría minimizar el riesgo de sesgo con la mínima pérdida de información:

Los ensayos clínicos pragmáticos se reconocen por una de las siguientes características: 1. El efecto del tratamiento se evalúa en las condi-

1. Aleatorizar los pacientes a cada grupo de tratamiento, y medir las variables resultado sólo en el

ciones más experimentales posibles.

2. Los pacientes que se incluyen en estos ensayos clínicos son muy homogéneos.

ojo derecho

3. Las pautas de tratamiento son jas para todos los sujetos. 4. En estos ensayos se intenta que los sujetos que se

2. Aleatorizar los pacientes a cada grupo de tratamiento, y medir las variables resultado sólo en los

incluyen sean muy parecidos a los pacientes que

3. Aleatorizar los pacientes a cada grupo de tratamiento, y medir las variables resultado sólo en los

2 ojos, sumando los resultados de ambos

se ven en la práctica clínica.

5. En enmascaramiento de las diferentes opciones

2 ojos, analizándolos por separado

4. Aleatorizar los pacientes a cada grupo de tratamiento, y medir las variables resultado sólo en

terapéuticas juega un papel fundamental en la evaluación de la ecacia de los tratamientos.

los 2 ojos, analizándolos por separado, y usando

técnicas que permitan corregir la correlación entre

170.

Señale la armación incorrecta acerca de las ven-

los 2 ojos de la misma persona 5. Aleatorizar un ojo de cada individuo al tratamien -

tajas e inconvenientes de las diferentes técnicas de muestreo:

to experimental y el otro al grupo control, y medir las variables resultado de forma independiente  para cada ojo

1. Si la población está ordenada, el muestreo siste mático asegura una cobertura de unidades de to -

173.

dos los tipos 2. El muestreo estraticado tiende a asegurar que la

Los diversos tipos de estudios cientícos tienen asignados diferentes niveles de evidencia cientíca

según la calidad de los mismos. En base a estas evidencias se establecen distintos grados de recomendación según la evidencia sea mejor o peor, y vaya a favor o en contra de la administración de un determinado medicamento en según qué circunstancias. Acerca de las escalas de gradación de la

muestra represente adecuadamente a la población en función de la presencia de una o unas variables seleccionadas 3. El muestreo aleatorio simple requiere un listado completo y previo de la población 4. El error estándar generado en un muestreo aleatorio simple es mayor que el generado en el muestreo en etapas múltiples

evidencia cientíca, señale la respuesta correcta:

5. El muestreo sistemático es, en la práctica, más

sencillo que el muestreo aleatorio simple

171.

1. La evidencia aportada por estudios de tipo descriptivo es de grado III, lo que supone que existe cierta evidencia para recomendar (o no recomen dar) la administración de ese fármaco en la situa -

En un estudio sobre las causas del cáncer de mama se escogió un grupo de mujeres postmenopáusicas con la enfermedad y un grupo de mujeres libres de la enfermedad, comparándose los antecedentes de paridad, enfermedades mamarias previas y administración de estrógenos en la menopausia. ¿Cómo

ción que se está evaluando.

2. Un estudio de cohortes prospectivo se considera un nivel de evidencia tipo II-1b, lo cual supone un grado de recomendación A a favor de una determinada práctica.

3. Los ensayos clínicos randomizados y controlados se consideran un nivel de evidencia cientíca tipo

clasicaría este estudio?

II-1a, lo cual supone un grado de recomendación D en contra de una determinada práctica. 4. Cuando hablamos de un grado de recomendación

1. Ensayo clínico.

D, entendemos que existe adecuada evidencia

2. Cohortes prospectivas. 3. Cohortes retrospectivas. 4. Casos y controles.

cientíca para no adoptar la práctica médica que

estamos valorando.

-28-

EXAMEN ETMR 19/12

5. Los estudios de casos y controles se consideran un

de la muestra, y constituye la medida de posición

nivel de evidencia cientíca tipo II-2b, y por tanto

más simple.

un grado de recomendación B donde existe cierta

3. La mediana es el punto medio del rango o recorrido. 4. En las distribuciones simétricas unimodales, la media, la mediana y la moda coinciden. 5. La moda o modas de la distribución son aquellos valores de la variable que tienen mayor frecuencia que sus adyacentes.

evidencia cientíca para adoptar una determinada  práctica médica.

174.

Dos variables presentan un coeciente de correlación (r) de 0,35, que es estadísticamente signica tivo (p50.000/ L de pre dominio polimorfonuclear, con cristales de urato. 5. Viscosidad alta, leucocitosis 20.000 L de predo minio polimorfonuclear, con cristales en aguja.

182.

¿Cuál de los siguientes procedimientos diagnósticos se debe utilizar en primer lugar, en un caso de sacroileítis aguda con ebre ondulante, y esplenomegalia?

186.

Paciente de 45 años, HLAB-27 positivo y diagnosticado de espondilitis anquilopoyética de 6 años de evolución. ¿Cuál de los siguientes hallazgos de laboratorio será el más frecuente, además de es-

1. Rectoscopia y biopsia. 2. Estudio anatomopatológico del bazo. 3. Tomografía axial computarizada.

pecíco?

4. Hemocultivo en medio de Ruiz-Castañeda.

1. 2. 3. 4.

5. Punción articular y cultivo en medio de Lowestein.

Proteinuria de rango nefrótico. Elevación de la fosfatasa alcalina. Incremento de IgA sérica. Elevación de velocidad de sedimentación globular (VSG) y PCR.

183.

5. Anticuerpos antinucleares y factor reumatoide positivos.

¿Cuál de las siguientes armaciones sobre el trata -

miento del LES NO es correcta? 187.

1. A veces la artritis lúpica responde a antipalúdicos. 2. Las manifestaciones potencialmente mortales de ben ser tratadas con glucocorticoides.

L.B. es una mujer de 35 años con historia de 3 meses de evolución de dolores articulares afectándole manos, hombros y, en menor grado, rodi-

3. Las artritis, mialgias y ebre mejoran con AINE,

llas y pies. Reere rigidez matutina de 3 horas de

 pero no con corticoides. 4. La toxicidad de los AINE como el aumento de

evolución, astenia intensa hasta el encamamiento,

las enzimas hepáticas, la meningitis aséptica y la afectación renal son más frecuentes en el LES. 5. En los casos más severos se utilizan bolos de cor -

A la exploración presenta tumefacción articular dolorosa en las articulaciones interfalángicas

sudoración nocturna y pérdida de 3 kg de peso.

proximales (IFP), metacarpo-falángicas (MCF),

ticoides intravenosos.

muñecas, hombros, rodillas y articulaciones metatarsianas (MTF). Tiene adenopatías periféricas

cervicales y axilares indoloras no adheridas, así como un nódulo en la supercie de extensión del

184.

185.

Las crioglobulinas se han asociado a todos los siguientes EXCEPTO: 1. Macroglobulinemia de Waldenström. 2. Mieloma múltiple.

antebrazo izquierdo. Los análisis de laboratorio muestran VSG 55 mm/hora, FR 1:1280, Hb 10,5 g/dl y aumento de proteínas totales a expensas de hipergammaglobulinemia policlonal. Las radiografías de manos y muñecas mostraron osteopenia yuxtaarticular, así como erosiones marginales

3. Hepatitis crónica C.

precoces, afectando la 4ª IFP de la mano derecha.

4. Enfermedad de Reiter. 5. Artritis reumatoide.

Fue iniciado tratamiento con AINE, prednisona en dosis bajas (5 mg/día) y metotrexato (7,5 mg/semana). En esta paciente, la forma de presentación de la enfermedad induce a tratar precozmente y de forma agresiva la enfermedad. Por favor, señale cuál de los siguientes enunciados NO es correcto:

¿Qué características denen mejor el l íquido sino-

vial en una monoartritis aguda por ácido úrico? 1. Los pacientes femeninos de raza blanca cursan con sinovitis de carácter más persistente y enfer -

1. Viscosidad alta, leucocitosis 50.000/ L de pre -

los varones. 2. Los pacientes que presentan títulos altos de FR,

dominio polimorfonuclear, cristales con forma de aguja.

PCR y haptoglobina tienen un peor pronóstico.

-30-

EXAMEN ETMR 19/12

3. La presencia de nódulos subcutáneos, como en el

ma básicamente cardiaco.

caso descrito, así como la de erosiones radiológicas, es índice de severidad pronóstica. 4. El patrón de presentación de la enfermedad no parece predecir el desarrollo de deformidades. 5. Aproximadamente el 40% de los pacientes con artritis reumatoide cursan con un proceso inamato rio agudo que remite sin deformidades mayores.

188.

3. Probablemente padezca un LES, pero debemos tener en cuenta que la plaquetopenia puede ser secundaria a la heparina de bajo peso mole-cular.

4. Probablemente padezca LES, pero no podemos hacer el diagnóstico, pues necesitamos 2 criterios clínicos y 2 analíticos. 5. La paciente padece un lupus ANA negativo.

Varón de 50 años sin antecedentes de interés que desde hace 4 meses presenta cefalea, pérdida de memoria y en la última semana se ha añadido disminución del nivel de conciencia y varios accidentes isquémicos transitorios en territorios de ambas arterias cerebrales medias. En la analítica destacan solo una VSG elevada, siendo las serologías para sílis, VIH, CMV, herpes simple, varicela y Borrelia negativas así como los ANA y ANCA. Radiografía de tórax, ecocardiograma y estudio Doppler de troncos supraaórticos sin alteraciones. La TC y la RMN objetivan múltiples infartos en corteza y sustancia blanca. La angiografía no mostró lesiones. En la biopsia cerebral se evidenció afectación de arteriolas con lesiones vasculíticas granulomatosas. El diagnóstico que Ud. sospecha en este paciente es:

190.

La queratodermia blenorrágica, típica lesión cutánea de la forma postvenérea de la artritis reactiva, es histológicamente indistinguible de las lesiones cutáneas de: 1. LES. 2. Síndrome de Behçet. 3. Psoriasis. 4. Enfermedad inamatoria intestinal. 5. Vasculitis leucocitoclástica.

191.

En la artritis reumatoide pueden tener lugar manifestaciones pleuropulmonares, no obstante algunas pueden ser totalmente inespecícas. En la

siguiente lista hay cuatro manifestaciones que son propias de la enfermedad y una que, cuando se presenta, es debida a mera coincidencia por azar:

1. Enfermedad de Buerger. 2. Enfermedad de Wegener. 3. Enfermedad de Eales.

1. 2. 3. 4.

4. Vasculitis primaria del SNC. 5. Enfermedad de Churg-Strauss.

Fibrosis intersticial difusa. Nódulos pulmonares. Pleuritis. Bronquiolitis obliterante.

5. Ensema.

189.

Mujer de 40 años acude a nuestra consulta por artralgias, y reere que le ha salido eritema malar y

que le molesta mucho el sol, produciéndole enro jecimiento marcado. Hace dos semanas sufrió una fractura de maléolo peroneo sin desplazar, por lo que lleva un yeso y se pincha por vía subcutánea 40mg de enoxaparina sódica al día. A la exploración presenta eritema malar, y como antecedentes

192.

El tratamiento de elección de la granulomatosis de Wegener es: 1. Prednisona y metotrexate.

reere una pericarditis que le diagnosticó hace

2. Ciclofosfamida y prednisona.

poco su cardiólogo. En una analítica se evidenció una plaquetopenia (90.000 plaquetas/mm3. ¿Qué

3. Azatioprina y prednisona. 4. Plasmaféresis y ciclofosfamida.

podemos armar ante este caso?

5. Clorambucil.

1. La paciente presenta sin ninguna duda un LES,  pues la fotosensibilidad, eritema malar, pericarditis y plaquetopenia son 4 criterios diagnósticos de lupus, número suciente para hacer el diagnosti co.

193.

2. Con los datos que tenemos no podemos hacer el diagnostico de LES; lo más probable es que no

 padezca LES, y con sus antecedentes, debemos orientar la búsqueda diagnóstica hacia un proble-

-31-

Mujer de 47 años con dolor e hipersensibilidad, e hinchazón brusca del pabellón auricular derecho. En la anamnesis descubre episodios de artritis de pocos días de duración, que se resolvían espontáneamente, varios episodios de conjuntivitis y que una vez le diagnosticaron una iridociclitis. También cuenta tos no productiva y ronquera ocasional. En la exploración encuentra una nariz con de-

EXAMEN ETMR 19/12

1. Iniciar un alfa-bloqueante.

formidad en silla de montar, dolor a la palpación de la laringe y al auscultarla un soplo distólico en foco aórtico. Lo más probable es:

2. Iniciar terapia combinada de alfa-bloqueante más nasteride. 3. Iniciar nasteride.

4. Adenomectomía abierta. 1. 2. 3. 4. 5.

194.

Granulomatosis de Wegener. Artritis reactiva. Policondritis recidivante. Síndrome de Reiter. Artritis reumatoide.

5. Conducta expectante.

197.

Un paciente de 16 años presenta un episodio compatible con cólico nefrítico. Ha tenido varios episodios desde su infancia y ha precisado la realización de 2 URS para la resolución del proceso.

Un paciente de 32 años acude a su consulta por incremento de tamaño testicular en los últimos dos meses. A la exploración se palpa nódulo testicular derecho. La ecografía muestra imagen intraparenquimatosa testicular derecha con calcicacio nes en su interior. Señale la respuesta correcta: 1. Se debe realizar una orquiectomía inguinal. 2. Se debe realizar una biopsia transescrotal. 3. La presencia de calcicaciones indica buen pro -

1. En la RX simple se observarán imágenes litiásicas

múltiples.

nóstico. 4. Se debe realizar biopsia testicular contralateral. 5. Se debe realizar quimioterapia neoadyuvante.

2. Se tratarán probablemente de litiasis cálcicas, por lo que habrá que disminuir su ingesta.

3. Se trata de litiasis muy sensibles al tratamiento con LEOC.

4. Puede que encontremos cristales con forma hexagonal en el sedimento.

198.

5. Lo más probable es que no se asocie a ningún tipo

de alteración metabólica.

Un paciente de 78 años acude a la consulta tras detectársele un PSA de 5.3 en una analítica realizada para control del colesterol por su MAP. Al interrogatorio reere orinar con cierta dicultad,

195.

con chorro entrecortado. Se suele levantar una única vez durante la noche. Niega hematuria, incontinencia e impotencia sexual. A la exploración se palpa una próstata adenomatosa grado III/IV al tacto rectal. Se realiza una biopsia de próstata ecodirigida que es informada como adenocarcinoma

Un paciente de 60 años es diagnosticado casualmente en ecografía de masa renal derecha de 7 centímetros de diámetro. En la analítica se evidencian Creatinina 0.95, Na 138, K 4.1, GOT 293, GPT 324, FA 842, BT 0.95, Hb 14.2, Leucos 9600, Plaquetas 180000. En este caso:

de próstata Gleason 6 (3+3) con afectación de un

cilindro derecho. Señale la respuesta correcta:

1. En la TAC probablemente encontraremos metástasis hepáticas.

1. Se trata de un T1c y puede tratarse con cirugía. 2. Se trata de un T2a y puede tratarse con cirugía. 3. Se trata de un T1c y su tratamiento de elección es

2. Se trata de un síndrome de Wunderlich. 3. Se debe a la compresión hepática por el tumor al

el BHC.

ser del lado derecho. 4. Los niveles de función renal deben hacernos plantearnos una nefrectomía parcial a pesar del tamaño tumoral. 5. Los efectos hepatotóxicos del tumor suelen desaparecer tras la extirpación renal.

4. Se trata de un T2b y su tratamiento de elección es la radioterapia. 5. Se trata de un T1c y puede tratarse con quimioterapia.

199. 196.

Un paciente de 62 años acude a su consulta porque ha notado últimamente disminución de la potencia del chorro de la orina con goteo postmiccional y para el inicio de la micción. El paciente realiza un cuestionario IPSS en la consulta y obtiene una calicación de 3, lo que calica su sintomatología de

leve. Usted le propondrá como tratamiento:

-32-

Acude a nuestra consulta una paciente de 17 años de edad, sin alergias a medicamentos conocidas, que vive con sus abuelos en medio rural, con cuadro que consiste en: galactorrea, ausencia de menarquia, telarquia grado II y pubarquia grado II. Se realiza un estudio bioquímico que demuestra la existencia de unos niveles de PRL de 155 ng/mL, una FSH de 3,98 mUI/mL, una LH de 0,559 mUI/ mL y un estradiol de 132 pg/mL. Se le realiza a la paciente una Resonancia Nuclear Magnética, que

EXAMEN ETMR 19/12

202.

se informa como “hipósis de tamaño normal, con

nódulo de 2,5 mm, sugestivo de microadenoma”. El tratamiento de elección de los pacientes con microprolactinomas es: 1. Agonistas dopaminérgicos. 2. Cirugía transesfenoidal. 3. Radioterapia hiposaria.

María es una mujer de 25 años, que al comenzar el invierno debuta clínicamente con un cuadro de tos, con escasa producción de moco, y sibilancias. Al estudiar a la paciente, en las pruebas funcionales se detecta un patrón obstructivo, con mejoría tras la prueba broncodilatadora. En el estudio de sangre periférica, así como en del esputo, se detecta eosinolia. Señale la armación errónea acerca

del tratamiento del asma:

4. Antagonistas dopaminérgicos. 5. Octreotide. 1. Una opción adecuada para el asma leve persistente podría ser budesonida pautada más salbutamol

200.

a demanda. 2. En el asma leve intermitente, basta el empleo de  beta-2-agonistas en el momento de las crisis.

Paciente de 32 años sin antecedentes médicos de interés y sin tratamientos, derivada por sospecha de hipocalcemia. En estudio analítico se observan los

3. Para el asma por ejercicio, puede ser útil el uso de

cromoglicato antes del mismo.

siguientes resultados: calcio total: 8,1 mg/dl (8,410,2); fósforo: 4,6 mg/dl (2,5-4,5); 25 OH2 vitamina D: 32 ng/ml (normal mayor de 30); PTH 235 pg/ml (10-65); calciuria orina 24 horas: 54 mg/24h (100-300); función renal y resto de parámetros en la bioquímica normal. TSH 5,2 mUI/mL (normal:

4. A diferencia del EPOC, los anticolinérgicos tie nen un papel más relevante en el tratamiento del

asma que los corticoides inhalados. 5. Son efectos frecuentes de los corticoides inhalados la disfonía, la tos y la irritación faríngea.

0,4-4 mUI/mL) con T4L normal. Función gonadal normal. La paciente no reere sintomatología

de hipocalcemia, y presenta un fenotipo normal. Trousseau negativo. Sus familiares no presentan alteración del metabolismo fosfo-cálcico conocido. Se inicia suplementación con calcio y vitamina D vía oral, con mejoría en las cifras de calcio y fósforo y disminución en los niveles de PTH ¿Cuál es la patología más probable?

203.

¿Por qué en la atelectasia NO suele ser importante la desaturación de O2? 1. Broncoconstricción. 2. Hiperventilación. 3. Circulación colateral. 4. Vasoconstricción selectiva. 5. Cortocircuito a nivel central.

1. Décit de vitamina D.

2. Pseudohipoparatiroidismo tipo Ib. 3. Pseudohipoparatirodismo tipo Ia. 4. Pseudopseudohipoparatiroidismo.

204.

5. Hipoparatiroidismo idiopático.

201.

Un niño de 16 meses de edad consulta por presentar, en el curso de un cuadro catarral de vías altas,

Es traído a Urgencias un paciente con agudización de EPOC. En la exploración el paciente está estuporoso y la gasometría arterial es la siguiente: pH 7,18, PaCO2 90 mmHg, PaO2 41 mmHg, HCO3 29 mEq/l. Indique cuál es la medida que deberá tomar en primer lugar:

ebre, irritabilidad y llanto. Exploración física: Tª

39,6 ºC, tímpano derecho hiperémico y abombado, faringe enrojecida con exudado amarillento y

1. Broncodilatadores de acción larga. 2. Intubación y ventilación mecánica. 3. Ventilación mecánica no invasiva. 4. Corticoides inhalados.

secreción nasal abundante. Señale la armación

FALSA en relación con este caso:

5. Antibioterapia. 1. La mayor incidencia de esta enfermedad se presenta entre los 6 meses y los 3 años de edad. 2. La hipertroa adenoidea puede ser un factor pre disponente. 3. La producción de betalactamasas por parte del

205.

neumococo es cada vez más frecuente. 4. La vía de propagación más frecuente es la tubári -

ca. 5. En condiciones normales, la mucosa que tapiza las cavidades del oído medio es estéril.

-33-

Una mujer de 29 años presenta un patrón intersticial con presencia de múltiples quistes en la TAC de tórax y un derrame pleural quiloso. Con respecto a la patología que presenta, ¿cuál es la armación correcta?

EXAMEN ETMR 19/12

1. Se describe casi exclusivamente en pacientes fumadores. 2. Se puede diagnosticar en el lavado broncoalveolar. 3. El tratamiento de elección es la medroxiprogesterona. 4. Se asocia con lesiones óseas líticas en huesos planos. 5. Predomina en campos superiores.

209.

Con respecto a la histiocitosis de células de Langerhans, ¿cuál de las siguientes armaciones no es

cierta? 1. La presencia de células de Langerhans en el lavado broncoalveolar o en la biopsia pulmonar es  patognomónica de la enfermedad. 2. Es una enfermedad casi exclusiva de fumadores. 3. Es muy característica la presencia simultánea de

206.

lesiones quísticas y nódulos pulmonares. 4. En el estudio funcional respiratorio podemos encontrar volúmenes elevados. 5. La principal medida terapéutica es el abandono del tabaco.

Panadero de 20 años que, a la una o dos horas de comenzar su trabajo, empieza con disnea y ruidos de pecho. En la espirometría tan sólo hay afectación de la pequeña vía aérea (MEF50 <

50%). De la siguiente medicación inhalada, ¿cuál será la más indicada como tratamiento de mantenimiento? 210. 1. Budesonida. 2. Cromoglicato sódico.

3. Bromuro de ipratropio. 4. Beclometasona. 5. Salmeterol.

207.

Varón de 58 años, fumador de 30 cigarrillos/día, con disnea de esfuerzo desde hace 8 meses. En el estudio de su función pulmonar se encuentran los siguientes valores: GAB con pH de 7,39, PaO2 de 70, PaCO2 de 36; Espirometría con CVF de 50%, VEF1 de 37% e índice VEF1/CVF de 74% sin modicación signicativa tras inhalación de broncodilatadores y Pletismografía con aumento de VR y de la CPT. De los siguientes diagnósticos, ¿cuál explicaría estos resultados?

Varón de 55 años que acude a su policlínica del Servicio de Neumología porque en un chequeo rutinario de su empresa se le ha detectado un derrame pleural en una radiografía de tórax. Trabaja como vigilante de un almacén desde hace diez años, pero anteriormente fue trabajador de la construcción; está asintomático. Se repite la radiografía de tórax, en la cual se observan opacidades lineales en los campos pulmonares inferiores, pérdida de nitidez del borde cardiaco, engrosamientos calcicados en la pleura diafragmática y borramiento del seno costofrénico derecho por un derrame pleural. En la espirometría se observa una disminución del volumen residual, de la capacidad vital y de la capacidad pulmonar total; el FEV1 es normal. ¿Cuál sería su diagnóstico? 1. 2. 3. 4.

1. Alveolitis alérgica extrínseca. 2. Tromboembolismo pulmonar de repetición. 3. Fibrosis pulmonar.

Silicosis. Mesotelioma. Asbestosis. Sarcoidosis.

5. Fibrosis pulmonar idiopática.

4. Ensema pulmonar.

211.

5. Neumoconiosis.

208.

Varón de 50 años que consulta por somnolencia diurna marcada. Presenta historia de ronquidos de años de evolución, es hipertenso y discretamen-

¿Cuál de los siguientes signos dene la crisis de

te obeso. Se le realiza un estudio polisomnográco simplicado que resulta negativo. ¿Cuál sería su

riesgo vital?

actitud en este caso?

1. Hipercapnia.

1. No realizar más estudios pues no presenta apnea

2. Pico de ujo espiratorio menor del 30% del teó -

rico. 3. Respiración paradójica.

2. 3.

4. Taquicardia mayor de 150 lpm. 5. Hipoxemia refractaria a tratamiento.

4. 5.

-34-

obstructiva del sueño. Realizaría una polisomnografía completa. Pautaría tratamiento inmediato con presión positiva continua de la vía aérea. Recomendaría al paciente sedación nocturna con  benzodiacepinas. Indicaría uvulopalatofaringoplastia.

EXAMEN ETMR 19/12

212.

Paciente varón de 7 años de edad que es traído a su consulta, según reeren sus padres, por ser ronca dor importante, incluso con pausas respiratorias. Durante el día el niño se encuentra bien, aunque en los últimos meses ha disminuido su rendimien-

cia de un soplo pansistólico en foco mitral. Se realiza un ECG que resulta normal. Se solicita una ecocardiografía, que revela la existencia de una insuciencia mitral degenerativa, con prolapso del

velo posterior por rotura de las cuerdas tendinosas. Se realiza estudio de la función ventricular, presentando un FEV del 50%. El cateterismo también informa de la ausencia de lesión de las arterias coronarias. Dadas las características clínicas y el grado de disfunción ventricular parece claramente indicado el tratamiento quirúrgico, por lo que el tratamiento de elección sería:

to escolar por décit de atención y dicultad para

la concentración. Le realiza una polisomnografía que pone de maniesto la existencia de un IAR de

28, siendo la mayoría de las apneas de origen obstructivo. Indique cuál de los siguientes considera el tratamiento de elección: 1. Amigdalectomía. 2. CPAP nocturno.

1. Comisurotomía.

3. Dispositivo de avance mandibular.

2. Dilatación con balón.

4. Osteotomía máxilo-mandibular.

3. Cirugía conservadora y reparadora de la válvula. 4. Sustitución de la válvula.

5. Uvulopalatofaringoplastia.

5. Trasplante cardíaco.

213.

Para hacer frente de forma ecaz a infecciones por

gérmenes intracelulares, que pueden escapar a la acción de las inmunoglobulinas, así como para resolver infecciones de determinados agentes, como virus u hongos, que difícilmente pueden ser erra-

215.

¿Cuál es, entre las siguientes, la explicación más probable para el cuadro de un paciente de 75 años, hipertenso, con disnea desde hace cuatro meses, sin soplos, con hipertroa ventricular izquierda en el

dicados por la acción de las inmunoglobulinas (de

ECG y silueta cardíaca normal en la radiografía?

forma aislada o conjunta a la del sistema de complemento), tenemos la inmunidad celular. Respecto a las respuestas de inmunidad celular, indique

1. Disfunción diastólica crónica del V.I. por hiper -

tensión.

la armación CORRECTA:

2. Disfunción sistólica crónica del V.I. por hiperten-

sión. 1. Las células NK siempre expresan, constitutiva-

3. Insuciencia mitral por dilatación del anillo val -

vular. 4. Infarto de miocardio antiguo sin onda “q”.

mente, el receptor de anidad media para IL-2.

2. Los linfocitos colaboradores TH1 frenan las reacciones de inmunidad celular una vez vencida la infección. 3. Los linfocitos colaboradores TH2 controlan y potencian las reacciones de inmunidad celular. 4. En su proceso de maduración en el timo, los timocitos que solo interaccionan con el HLA-I se

5. Disfunción sistólica crónica del V.I. por miocar -

diopatía dilatada.

216.

convierten en LT-CD4.

¿Cuál de las siguientes alteraciones del ritmo tiene peor pronóstico?

5. En su proceso de maduración en el timo, los timocitos capaces de interaccionar con el HLA-II se 1. Bloqueo incompleto de rama derecha. 2. Bloqueo completo de rama izquierda. 3. Bloqueo de rama derecha con hemibloqueo anterior. 4. Bloqueo aurículoventricular tipo Mobitz II. 5. Bloqueo aurículoventricular tipo Wenckebach.

convierten en LT-CD8.

214.

Paciente varón de 63 años de edad, hipertenso y con alergia a metamizol, que presenta clínica de disnea progresiva y sensación de cansancio desde hace 4 meses. El paciente está a punto de jubilarse en su empresa y había atribuido su sintomatología al estrés que tiene en su trabajo para dejarlo todo listo antes de marcharse. Hoy acude a urgencias porque ayer por la noche cuenta episodio de ortopnea, de dos almohadas, y esta mañana al le-

217.

vantarse no ha podido hacer nada (“ni atarse los

cordones de los zapatos”) sin fatigarse en extremo. En la exploración de Urgencias destaca la existen-

-35-

Varón de 78 años sin FRCV y sin enfermedades importantes que presenta cuadro de dolor torácico y síncope en la última semana. En la EF destaca soplo sistólico irradiado a carótidas que borra segundo tono y en ECG aparecen datos de HVI. ¿Cuál sería la actitud más apropiada en este paciente?

EXAMEN ETMR 19/12

1. Ergometría. 2. Ecocardiograma.

fía demuestra oclusión crónica y larga de la ilíaca derecha. El eje ilíaco izquierdo es normal. ¿Qué actitud sería la más adecuada?

3. Holter para descartar BAVC como causa del sín -

cope. 4. Coronariografía por angina inestable y síncope.

5. Ergometría con isótopos por alteración en la repolarización.

1. By-pass femorofemoral. 2. By-pass aortofemoral mediante laparotomía media 3. By-pass femoropoplíteo con vena safena invertida

Mujer de 63 años hipertensa, que presenta dolor precordial opresivo y sudoración de dos horas de evolución. En la exploración física destaca una TA

4. Tratamiento médico dado el elevadísimo riesgo quirúrgico. 5. Perfusión de prostaglandinas y demorar la cirugía hasta que pase un año tras el último infarto de miocardio.

 bajo anestesia raquídea.

218.

de 170/95 mmHg, y el electrocardiograma (ECG)

muestra ritmo sinusal a 105 lpm y elevación de segmento ST de 2 mm en derivaciones precordiales desde V1 hasta V4. ¿Cuál de las siguientes medidas NO es adecuada? 221.

El síndrome X consiste en:

1. Reposo en cama y monitorización ECG durante

dos o tres días.

1. Angina con ergometría negativa y coronarias normales. 2. Angina con ergometría positiva y coronarias normales. 3. Angina con ergometría negativa y lesiones coronarias de un vaso. 4. Angina con ergometría positiva y coronarias de tres vasos. 5. Angina con ergometría negativa y enfermedad del tronco de la coronaria izquierda.

2. Alivio del dolor con opiáceos y/o nitroglicerina.

3. Administrar sin demora aspirina y tratamiento trombolítico, una vez descartada la existencia de contraindicaciones al mismo. 4. Administrar verapamilo o diltiacem como vasodilatadores coronarios. 5. Iniciar tratamiento con betabloqueantes.

219.

Varón de 26 años que consulta por episodios de palpitaciones ocasionales mal toleradas. La EF es normal, así como el ECG basal. En un Holter de 24 h presenta extrasístoles ventriculares muy frecuentes con origen en el tracto de salida de VD. ¿Cuál es la actitud más adecuada?

222.

1. Iniciar tratamiento betabloqueante. 2. Iniciar tratamiento calcioantagonista. 3. Descartar cardiopatía estructural mediante ecocardiograma y/o resonancia magnética cardiaca. 4. Iniciar tratamiento con amiodarona. 5. Seguimiento clínico.

220.

Paciente varón de 64 años, hipertenso y diabético tipo II de 8 años de evolución en tratamiento con insulina. Afecto de una cardiopatía isquémica que debutó hace 2 años en forma de infarto anterolateral extenso. Un ecocardiograma reciente ha mostrado un ventículo izquierdo ligeramente dilatado con una función sistólica del ventrículo izquierdo del 41%. En una analítica de unos meses antes se había constatado un ltrado glomerular de 39

ml/min. El paciente recibe ácido acetilsalicílico, atenolol y enalapril. Acude a una visita de control al cardiólogo donde destaca presión arterial de 162/96 mmHg. El electrocardiograma muestra bradicardia sinusal a 42 lpm, con ondas T picudas en cara anterior y un QRS de 130 mseg. El paciente no presenta angina.¿ Cuál de las siguientes actitudes sería la más apropiada?

Varón de 72 años. Factores de riesgo cardiovascular HTA, hipercolesterolemia y ex-fumador. EPOC severo con O2 domiciliario. Insuciencia

renal crónica prediálisis. Historia cardiológica: IAM hace 1 año, reinfarto a los 6 meses y realización de ACTP + stent en DA y CD con buen

1. Citar al paciente en un año dado que está estable

de su cardiopatía. 2. Añadir al tratamiento 1 comprimido de digoxina al día. 3. Solicitar una analítica con niveles de potasio.

resultado angiográco. FEVI 40%. Consulta por dolor de reposo y alteraciones trócas importantes

en pie derecho. En la exploración no se detectan pulsos en el eje derecho con existencia de pulsos distales en el izquierdo. Los índices T/B son: en el derecho 0,3 y en el izquierdo 0,9. La arteriogra-

4. Solicitar un angioTAC de arterias coronarias. 5. Debería implantarse un marcapasos denitivo

DDD.

-36-

EXAMEN ETMR 19/12

223.

Paciente de 38 años que consulta por disnea y palpitaciones en relación con esfuerzos vigorosos, en la exploración tiene un soplo sistólico rudo que aumenta con la maniobra de Valsalva y en el estudio eco-Doppler presenta un engrosamiento severo de las paredes del ventrículo izquierdo con un gradiente sistólico en el tracto de salida del ventrículo izquierdo de 20 mmHg. ¿Cuál de las siguientes

5. Deberá evitar tener despertador con sonido.

226.

Señale la respuesta correcta respecto a la vacunación contra la infección por HPV en España: 1. La vacuna bivalente protege también contra condilomas genitales.

armaciones es cierta?

2. La población diana son mujeres entre 14 y los 16

años.

1. La disnea está en relación con la severidad del gradiente dinámico en el tracto de salida del ven -

3. La vacunación se realiza en cuatro dosis (0, 3, 6 y 12 meses). 4. La vacuna tetravalente conere protección contra

trículo izquierdo. 2. La disnea está en relación con la mayor rigidez de

la pared de ventrículo izquierdo.

los serotipos: 6, 11, 16 y 18. 5. La administración de la dosis de recuerdo es sub-

3. La disnea está en relación con disfunción sistólica

del ventrículo izquierdo.

cutánea.

4. La disnea está en relación con la regurgitación

valvular mitral. 5. La disnea es un síntoma muy infrecuente en estos  pacientes.

224.

227.

1. Existencia en el receptor de anticuerpos contra moléculas HLA presentes en las células del donante. 2. Activación de linfocitos T citotóxicos de memoria. 3. Incremento de factores de quimiotaxis como consecuencia de la activación del complemento por vía alternativa, desencadenada en las membranas

Señale cuál de las siguientes es correcta en relación a las taquicardias supraventriculares: 1. El utter auricular es más frecuente en mujeres

que en hombres. 2. Las taquicardias auriculares son las arritmias su praventriculares más frecuentes.

3. Las taquicardias por reentrada intranodal son las únicas que responden a adenosina. 4. El tratamiento de elección de las taquicardias inta-

de hemodiálisis. 4. Hiperactivación de los neutrólos por la uremia

crónica. 5. Activación linfocitaria masiva tras la brusca elevación de eritropoyetina que se desarrolla tras el trasplante.

nodales en fase aguda es la ecainida. 5. El utter auricular se asocia a enfermedad pulmo-

nar obstructiva crónica.

225.

El rechazo hiperagudo de un trasplante renal se debe a:

Una chica de 22 años está siendo estudiada por episodios sincopales por lo que, entre otras pruebas, se le ha colocado un Holter-ECG de 24h. Mientras espera en el andén del tren, tras el pitido de este, sufre un nuevo síncope y en el dispositivo se registra una taquicardia ventricular helicoidal autolimitada. Cuando acude a consulta objetivamos en el ECG un intervalo QT corregido por frecuencia cardiaca de 520ms. En relación a la patología que sospecha, ¿cuál de las siguientes opciones cree que es falsa?:

228.

Un paciente sufre una caída de motocicleta con traumatismo cervical con lateralización brusca del cuello. Presenta décit por abducir el hombro y levemente para exionar el codo. El reejo bic -

cipital está disminuido. El resto de la exploración del miembro superior es normal. La lesión afecta a la raíz. 1. 2. 3. 4. 5.

1. Es una patología congénita.

C5. C6. C7. C5 y C6. C5, C6 y C7.

2. Deberíamos implantar un DAI lo más pronto po -

sible. 3. Hay una hipofunción de los canales de potasio. 4. Se debería iniciar tratamiento con beta-bloqueantes.

229.

Un varón de 63 años presenta ebre, tos, expecto-

ración herrumbrosa, dolor pleurítico en la región

-37-

EXAMEN ETMR 19/12

basal de hemitórax derecho y un inltrado de ca -

1. Hernia discal L5-S1.

racterísticas alveolares en LID. Inicia tratamiento con cefalosporinas de segunda generación, y a los

2. Insuciencia venosa de miembros inferiores.

3. Síndrome de cola de caballo. 4. Arteriosclerosis de miembros inferiores. 5. Estenosis del canal lumbar.

6 días comienza a presentar nuevamente ebre,

 junto con disnea de moderados esfuerzos. La actitud a seguir será: 232.

1. Cambiar el tratamiento a cefalosporinas de 3ª ge -

neración. 2. Es frecuente que en la primera semana de tratamiento de una neumonía existan episodios de em peoramiento clínico, por lo cual seguiremos con la misma pauta de tratamiento. 3. Se debe realizar Rx tórax, y en caso de que la disnea se deba a derrame pleural, haremos toracocentesis evacuadora paliativa. 4. Si existe derrame pleural, se realizará toracocen tesis diagnóstica. En caso de que se trate de un

¿Cuál es el animal que transmite habitualmente la hidatidosis al hombre? 1. Cerdo.

2. Gato. 3. Perro. 4. Ganado vacuno. 5. Cabra.

exudado, se colocará drenaje torácico.

233.

5. Si al realizar una Rx tórax se demuestra la presencia de un derrame pleural se colocará drenaje torácico en caso de glucosa en líquido pleural sea

Paciente mujer de 42 años de edad, con antecedente de cirugía de columna lumbar hace 10 años, con instrumentación quirúrgica, insuciencia venosa

crónica y toma de anticonceptivos, que acude a

View more...

Comments

Copyright ©2017 KUPDF Inc.
SUPPORT KUPDF